Stack Exchange Network

Stack Exchange network consists of 183 Q&A communities including Stack Overflow , the largest, most trusted online community for developers to learn, share their knowledge, and build their careers.

Q&A for work

Connect and share knowledge within a single location that is structured and easy to search.

Double slit experiment outcome changing by observing the slits

my question is as I believe quite simple since I'm new to physics. However here it is: if we take a double slit and constantly shoot helium atoms on it with a constant speed one by one we will see a certain interference pattern on a properly set up screen behind the double slit (scenario 1). If we now set up a detector that tells us which of the slits each atom is passing the pattern on the screen will change (scenario 2), so far so good. But if we were to set up the same detector but without looking at what it tells us and the information wouldn't be saved what would happen? So the machine detects where each atom is passing but nobody looks at the information and it goes basically instantly lost, would we see the pattern from scenario 1 or 2? (Btw I'm not a native English speaker so I apologize if I used unprofessional terms at some point)

  • quantum-mechanics
  • double-slit-experiment

user21457835's user avatar

  • $\begingroup$ Hint: in your scenario 2 that you seem to accept, will the interaction between the detector and the atom change in any way depending of whether a human learns about the new state of the detector after the interaction has taken place? $\endgroup$ –  Marius Ladegård Meyer Commented Mar 22, 2023 at 16:53

4 Answers 4

I hope the following explanation isn't too advanced for you. The gist is - in real quantum mechanics you don't need to philosophize about what is and is not a measurement. An interaction with a detector ruins the particles' ability to make an interference pattern.

Interaction with a detector introduces an uncontrolled random phase on the wavefunction which makes the interference pattern disappear. A more sophisticated understanding of quantum mechanics recognizes that there is no distinct moment that can be called a "measurement." Instead, when you have your particles interact with something like a detector, "coherence" (defined as having a well-defined, reproducible phase between two parts of a wave function) is lost, and although the particle remains in a superposition, the two halves of the wave function dont consistently add and subtract from eachother the way they did when they were making the interference pattern. And the result of the experiment is the same as if you had just sent particles through either particular slit one at a time and averaged the results.

AXensen's user avatar

  • $\begingroup$ "n real quantum mechanics you don't need to philosophize about what is and is not a measurement".I disagree.The measurement problem is a real problem because it affects the interpretation of reality. $\endgroup$ –  appliedSciences Commented Mar 22, 2023 at 18:22
  • 2 $\begingroup$ I recommend this lecture series - it's where I get most of my understanding of loss of coherence and how (particularly for answering a question like this) it's a more useful way of understanding "measurement" ocw.mit.edu/courses/… To some extent there remains a philosophical question that goes something like "what is a conscious entity and when does it decide what part of the wave function to be aware of" but for the purpose of this question there is no such issue. $\endgroup$ –  AXensen Commented Mar 22, 2023 at 18:26
  • $\begingroup$ PBS spacetime seems to have propagated this misunderstanding that the detector adds a random phase. That's not necessary (see physics.stackexchange.com/questions/204100/… , e.g.). Also, if it did add a random phase, then we would expect to see a smeared-out interference pattern, and not two bright lines like we actually do. $\endgroup$ –  A_P Commented Dec 2, 2023 at 2:47
  • $\begingroup$ @A_P Not sure about pbs spacetime, but I linked a lecture series from a nobel prize winner where this idea is presented (although this was a long time ago so I no longer remember where in the lecture). But your second sentence is completely wrong. If the distribution of landing positions through each single slit do not overlap, there will be no interference pattern . Just look at the wiki for the double slit experiment where it compares the single slit result to the double. You do not get two separate bright lines when you add the detector en.wikipedia.org/wiki/Double-slit_experiment $\endgroup$ –  AXensen Commented Dec 2, 2023 at 22:21
  • $\begingroup$ @AXensen Do you mean my third sentence? Yes, you're right that if there are actually detectors there, then there will be no interference pattern at the screen whether or not they add a phase shift. What I mean to say is that if they only add a phase shift (and don't entangle with the electron) then you will not reproduce the two bands; you will merely smear out the interference pattern. Are you certain that this Nobel laureate really claims that the decoherence here is caused by adding a random phase? Because it's really not necessary, as my link shows (and as all QC students learn). $\endgroup$ –  A_P Commented Dec 4, 2023 at 2:30

The type of experiment you are thinking are known as Quantum Erasure experiments:

https://en.wikipedia.org/wiki/Quantum_eraser_experiment

The short answer is, yes, you can create an experiment where each photon goes through slits and gets tagged on which slit it passed. If you look at the tag, the interference pattern disappears. However, you can "untag" the photon and you will restore the interference pattern.

In my experience, though, you need to look at the details of each experiment: you can't make a general conclusion. If you really look at the setup of each experiment, you find that it is less "surprising" than the abstract made it look like.

You can see these videos for a better discussion:

https://www.youtube.com/watch?v=l8gQ5GNk16s&ab_channel=Fermilab https://www.youtube.com/watch?v=RQv5CVELG3U&ab_channel=SabineHossenfelder

Hope it helps!

Carcassi's user avatar

This question can be answered without reference to human interaction, see the reference below for the experimental demonstration using photons.

The general rule for double slit interference is: There will be interference UNLESS there is the possibility, in principle, for determining which-slit information. It does not matter whether you know which slit the particle goes through, it is enough to eliminate the interference that you could have obtained this information.

In the cited experiment, a polarizer is placed in front of each of the 2 slits. When the polarizers are oriented parallel, the traditional interference pattern appears (there is interference). When the polarizers are crossed (orthogonal), there is no interference pattern. (Note that you can vary the angle between the polarizers from 0 to 90 degrees and get a mixture of more or less of the interference pattern.)

In this scenario with slit filters crossed (90 degrees apart), it would be possible to further filter the particles hitting the detection screen to determine which slit the particle went through. Therefore no interference occurs. It matters not that you don't actually obtain this information, it is enough that you could have. Note that in this experiment, you don't need to consider that the human does or doesn't look at the results.

Experiment: https://sciencedemonstrations.fas.harvard.edu/files/science-demonstrations/files/single_photon_paper.pdf

Theory only: https://arxiv.org/abs/1110.4309

DrChinese's user avatar

In general you do not need to make the conscious observation to destroy the pattern, interaction with the light will be enough of an interaction to alter the particle path after the slits ... the slits are no longer part of the wave function.

Interference patterns whether they be photons themselves, electrons or even the Buckyballs ..... are all a result of forces that favour the energy (photons) or masses (electrons, particles) to move to certain areas (bright spots) and not other areas (dark lines). The responsible force is the EM (electromagnetic force) which we say is governed by the EM field which is everywhere.

For photons and electrons the EM field is already active even before the photon or electron leaves the atom, i.e the excited electron in the atom is fully interacting with the EM field over distances. For buckyballs the EM field interaction is more subtle.

PhysicsDave's user avatar

  • $\begingroup$ The easiest double slit experiment to set up is with light - not charged particles. Here they do it successfully with buckyballs (C60) from an oven at 900K documents.epfl.ch/groups/i/ip/ipg/www/2016-2017/… . But moreover, I think this answer is just wrong - why do you think it matters whether we use voltages or gravity or just velocity? $\endgroup$ –  AXensen Commented Mar 23, 2023 at 8:32
  • $\begingroup$ That's a good question. Another article also proves your point. arxiv.org/pdf/1402.1867.pdf . In your referenced paper there is an ionization step (!), this reference is cleaner in that the observation involves no ionization. I'll make some changes above. $\endgroup$ –  PhysicsDave Commented Mar 23, 2023 at 16:27
  • $\begingroup$ @AndrewChristensen see comment above, forget to add name. $\endgroup$ –  PhysicsDave Commented Mar 24, 2023 at 15:27
  • $\begingroup$ I only brought up the massless particles to make the broader point that this answer is totally wrong about what causes interference. The dark lines in interference are not in any way due to forces pushing the particles out of those regions. It is because of two halves of the wavefunction cancelling out because they have opposite phase. This is particularly clear in the case of photons (no forces whatsoever act on photons). I'll link to one source explaining the effect pressbooks.online.ucf.edu/phy2053bc/chapter/… $\endgroup$ –  AXensen Commented Mar 24, 2023 at 16:11
  • $\begingroup$ @AndrewChristensen If you can agree that the wave functions are virtual then I think we are in agreement. I'm a Feynman fan, every photon's path is pre-determined ... and this can only be due to the EM field. The EM field is full of virtual as well as real photons .... and the EM field guides everything (photons, electrons, buckyballs). $\endgroup$ –  PhysicsDave Commented Mar 24, 2023 at 19:43

Your Answer

Sign up or log in, post as a guest.

Required, but never shown

By clicking “Post Your Answer”, you agree to our terms of service and acknowledge you have read our privacy policy .

Not the answer you're looking for? Browse other questions tagged quantum-mechanics double-slit-experiment or ask your own question .

  • Featured on Meta
  • We've made changes to our Terms of Service & Privacy Policy - July 2024
  • Introducing an accessibility dashboard and some upcoming changes to display...

Hot Network Questions

  • My first cryptic crossword
  • Is it OK to call a person "sempai" who is actually much younger than you?
  • When are these base spaces isomorphic?
  • How can I pass a constructed image path to \lettrine?
  • What is the anti-trust argument made by X Corp's recent lawsuit against advertisers that boycotted X/Twitter
  • How does one find historic prices for goods?
  • Why is “water takes the steepest path downhill” a common approximation?
  • Word/phrase for avoiding a task because it has a lot of unknowns
  • How different is Wittgenstein's Language game from Contextuality?
  • What is this notion of differentiability?
  • Fantasy book in which a joker wonders what's at the top of a stone column
  • Double expansion option for a single card
  • What 'technical grounds' did the Spanish supreme court invoke to not apply the amnesty to Puigdemont?
  • Branching on whether a color is defined using xcolor or xcolor-material?
  • Why is my single speed bike incredibly hard to pedal
  • What happens if your child sells your car?
  • What does "No camping 10-21" mean?
  • How to proceed if my manager is ghosting me?
  • Schengen visa expires during my flight layover
  • Can right shift by n-bits operation be implemented using hardware multiplier just like left shift?
  • I have two identical wi-fi drivers installed?
  • 4 X 4/0 or 3 X 4/0+2/0 for manufactured home service entrance
  • Why did Jesus give Pilate "no answer" to the question “Where are You from?” (John 19:9)?
  • Four Year Old Pushing and Positive Discipline Techniques

double slit experiment quora

Reset password New user? Sign up

Existing user? Log in

Double-slit Experiment

Already have an account? Log in here.

Plane wave representing a particle passing through two slits, resulting in an interference pattern on a screen some distance away from the slits. [1] .

The double-slit experiment is an experiment in quantum mechanics and optics demonstrating the wave-particle duality of electrons , photons , and other fundamental objects in physics. When streams of particles such as electrons or photons pass through two narrow adjacent slits to hit a detector screen on the other side, they don't form clusters based on whether they passed through one slit or the other. Instead, they interfere: simultaneously passing through both slits, and producing a pattern of interference bands on the screen. This phenomenon occurs even if the particles are fired one at a time, showing that the particles demonstrate some wave behavior by interfering with themselves as if they were a wave passing through both slits.

Niels Bohr proposed the idea of wave-particle duality to explain the results of the double-slit experiment. The idea is that all fundamental particles behave in some ways like waves and in other ways like particles, depending on what properties are being observed. These insights led to the development of quantum mechanics and quantum field theory , the current basis behind the Standard Model of particle physics , which is our most accurate understanding of how particles work.

The original double-slit experiment was performed using light/photons around the turn of the nineteenth century by Thomas Young, so the original experiment is often called Young's double-slit experiment. The idea of using particles other than photons in the experiment did not come until after the ideas of de Broglie and the advent of quantum mechanics, when it was proposed that fundamental particles might also behave as waves with characteristic wavelengths depending on their momenta. The single-electron version of the experiment was in fact not performed until 1974. A more recent version of the experiment successfully demonstrating wave-particle duality used buckminsterfullerene or buckyballs , the \(C_{60}\) allotrope of carbon.

Waves vs. Particles

Double-slit experiment with electrons, modeling the double-slit experiment.

To understand why the double-slit experiment is important, it is useful to understand the strong distinctions between wave and particles that make wave-particle duality so intriguing.

Waves describe oscillating values of a physical quantity that obey the wave equation . They are usually described by sums of sine and cosine functions, since any periodic (oscillating) function may be decomposed into a Fourier series . When two waves pass through each other, the resulting wave is the sum of the two original waves. This is called a superposition since the waves are placed ("-position") on top of each other ("super-"). Superposition is one of the most fundamental principles of quantum mechanics. A general quantum system need not be in one state or another but can reside in a superposition of two where there is some probability of measuring the quantum wavefunction in one state or another.

Left: example of superposed waves constructively interfering. Right: superposed waves destructively interfering. [2]

If one wave is \(A(x) = \sin (2x)\) and the other is \(B(x) = \sin (2x)\), then they add together to make \(A + B = 2 \sin (2x)\). The addition of two waves to form a wave of larger amplitude is in general known as constructive interference since the interference results in a larger wave.

If one wave is \(A(x) = \sin (2x)\) and the other is \(B(x) = \sin (2x + \pi)\), then they add together to make \(A + B = 0\) \(\big(\)since \(\sin (2x + \pi) = - \sin (2x)\big).\) This is known as destructive interference in general, when adding two waves results in a wave of smaller amplitude. See the figure above for examples of both constructive and destructive interference.

Two speakers are generating sounds with the same phase, amplitude, and wavelength. The two sound waves can make constructive interference, as above left. Or they can make destructive interference, as above right. If we want to find out the exact position where the two sounds make destructive interference, which of the following do we need to know?

a) the wavelength of the sound waves b) the distances from the two speakers c) the speed of sound generated by the two speakers

This wave behavior is quite unlike the behavior of particles. Classically, particles are objects with a single definite position and a single definite momentum. Particles do not make interference patterns with other particles in detectors whether or not they pass through slits. They only interact by colliding elastically , i.e., via electromagnetic forces at short distances. Before the discovery of quantum mechanics, it was assumed that waves and particles were two distinct models for objects, and that any real physical thing could only be described as a particle or as a wave, but not both.

In the more modern version of the double slit experiment using electrons, electrons with the same momentum are shot from an "electron gun" like the ones inside CRT televisions towards a screen with two slits in it. After each electron goes through one of the slits, it is observed hitting a single point on a detecting screen at an apparently random location. As more and more electrons pass through, one at a time, they form an overall pattern of light and dark interference bands. If each electron was truly just a point particle, then there would only be two clusters of observations: one for the electrons passing through the left slit, and one for the right. However, if electrons are made of waves, they interfere with themselves and pass through both slits simultaneously. Indeed, this is what is observed when the double-slit experiment is performed using electrons. It must therefore be true that the electron is interfering with itself since each electron was only sent through one at a time—there were no other electrons to interfere with it!

When the double-slit experiment is performed using electrons instead of photons, the relevant wavelength is the de Broglie wavelength \(\lambda:\)

\[\lambda = \frac{h}{p},\]

where \(h\) is Planck's constant and \(p\) is the electron's momentum.

Calculate the de Broglie wavelength of an electron moving with velocity \(1.0 \times 10^{7} \text{ m/s}.\)

Usain Bolt, the world champion sprinter, hit a top speed of 27.79 miles per hour at the Olympics. If he has a mass of 94 kg, what was his de Broglie wavelength?

Express your answer as an order of magnitude in units of the Bohr radius \(r_{B} = 5.29 \times 10^{-11} \text{m}\). For instance, if your answer was \(4 \times 10^{-5} r_{B}\), your should give \(-5.\)

Image Credit: Flickr drcliffordchoi.

While the de Broglie relation was postulated for massive matter, the equation applies equally well to light. Given light of a certain wavelength, the momentum and energy of that light can be found using de Broglie's formula. This generalizes the naive formula \(p = m v\), which can't be applied to light since light has no mass and always moves at a constant velocity of \(c\) regardless of wavelength.

The below is reproduced from the Amplitude, Frequency, Wave Number, Phase Shift wiki.

In Young's double-slit experiment, photons corresponding to light of wavelength \(\lambda\) are fired at a barrier with two thin slits separated by a distance \(d,\) as shown in the diagram below. After passing through the slits, they hit a screen at a distance of \(D\) away with \(D \gg d,\) and the point of impact is measured. Remarkably, both the experiment and theory of quantum mechanics predict that the number of photons measured at each point along the screen follows a complicated series of peaks and troughs called an interference pattern as below. The photons must exhibit the wave behavior of a relative phase shift somehow to be responsible for this phenomenon. Below, the condition for which maxima of the interference pattern occur on the screen is derived.

Left: actual experimental two-slit interference pattern of photons, exhibiting many small peaks and troughs. Right: schematic diagram of the experiment as described above. [3]

Since \(D \gg d\), the angle from each of the slits is approximately the same and equal to \(\theta\). If \(y\) is the vertical displacement to an interference peak from the midpoint between the slits, it is therefore true that

\[D\tan \theta \approx D\sin \theta \approx D\theta = y.\]

Furthermore, there is a path difference \(\Delta L\) between the two slits and the interference peak. Light from the lower slit must travel \(\Delta L\) further to reach any particular spot on the screen, as in the diagram below:

Light from the lower slit must travel further to reach the screen at any given point above the midpoint, causing the interference pattern.

The condition for constructive interference is that the path difference \(\Delta L\) is exactly equal to an integer number of wavelengths. The phase shift of light traveling over an integer \(n\) number of wavelengths is exactly \(2\pi n\), which is the same as no phase shift and therefore constructive interference. From the above diagram and basic trigonometry, one can write

\[\Delta L = d\sin \theta \approx d\theta = n\lambda.\]

The first equality is always true; the second is the condition for constructive interference.

Now using \(\theta = \frac{y}{D}\), one can see that the condition for maxima of the interference pattern, corresponding to constructive interference, is

\[n\lambda = \frac{dy}{D},\]

i.e. the maxima occur at the vertical displacements of

\[y = \frac{n\lambda D}{d}.\]

The analogous experimental setup and mathematical modeling using electrons instead of photons is identical except that the de Broglie wavelength of the electrons \(\lambda = \frac{h}{p}\) is used instead of the literal wavelength of light.

  • Lookang, . CC-3.0 Licensing . Retrieved from https://commons.wikimedia.org/w/index.php?curid=17014507
  • Haade, . CC-3.0 Licensing . Retrieved from https://commons.wikimedia.org/w/index.php?curid=10073387
  • Jordgette, . CC-3.0 Licensing . Retrieved from https://commons.wikimedia.org/w/index.php?curid=9529698

Problem Loading...

Note Loading...

Set Loading...

Follow along with the video below to see how to install our site as a web app on your home screen.

Note: This feature may not be available in some browsers.

  • Classical Physics

The double-slit experiment with a pit in the screen

  • Thread starter Imya
  • Start date Jul 31, 2024
  • Tags Double-slit experiment Interference and diffraction Optics Wave-particle duality
  • Jul 31, 2024

double slit experiment quora

  • Research team improves fuel cell durability with fatigue-resistant membranes
  • Apex predators not a quick fix for restoring ecosystems, 20-year study finds
  • Building images photon-by-photon to increase the information content provided by microscopes
  • Aug 1, 2024

A PF Planet

Imya said: How does the distribution of light power between the pit and the rest of the flat screen depend on opening two slits or closing one slit? Five predictable experiments were previously described to formulate a question for the experiment #6.
The region A sees slit α and does not see slit β. The region B sees slit β and does not see slit α. This means that a straight ray from A falls on α, but not on β. A straight ray from B falls on β, but not on α.
  • Yesterday, 12:45 PM
Imya said: TL;DR Summary: How does the distribution of light power between the pit and the rest of the flat screen depend on opening two slits or closing one slit? Five predictable experiments were previously described to formulate a question for the experiment #6. But how can the power on the rest of the screen change due to the appearance of pits in it?
Nugatory said: To approach this problem correctly you will have to use wave methods consistently throughout:

Similar threads

  • Jan 31, 2022
  • Mar 11, 2022
  • Apr 23, 2023
  • Apr 5, 2022
  • Nov 15, 2023
  • Jun 3, 2023
  • Jul 9, 2021
  • May 22, 2020
  • Dec 25, 2021
  • Feb 10, 2022

Hot Threads

  • I   I'm calculating more energy out than I put in
  • B   Why 186,282?
  • I   How Does Friction Function in the Movement and Turning of a Car?
  • I   Factors that influence depth of field
  • B   Sine wave noise at different frequencies

Recent Insights

  • Insights   PBS Video Comment: “What If Physics IS NOT Describing Reality”
  • Insights   Aspects Behind the Concept of Dimension in Various Fields
  • Insights   Views On Complex Numbers
  • Insights   Addition of Velocities (Velocity Composition) in Special Relativity
  • Insights   Schrödinger’s Cat and the Qbit
  • Insights   The Slinky Drop Experiment Analysed

double slit experiment quora

  • Follow us on Facebook
  • Follow us on Twitter
  • Follow us on LinkedIn
  • Watch us on Youtube
  • Audio and video Explore the sights and sounds of the scientific world
  • Podcasts Our regular conversations with inspiring figures from the scientific community
  • Video Watch our specially filmed videos to get a different slant on the latest science
  • Webinars Tune into online presentations that allow expert speakers to explain novel tools and applications
  • Latest Explore all the latest news and information on Physics World
  • Research updates Keep track of the most exciting research breakthroughs and technology innovations
  • News Stay informed about the latest developments that affect scientists in all parts of the world
  • Features Take a deeper look at the emerging trends and key issues within the global scientific community
  • Opinion and reviews Find out whether you agree with our expert commentators
  • Interviews Discover the views of leading figures in the scientific community
  • Analysis Discover the stories behind the headlines
  • Blog Enjoy a more personal take on the key events in and around science
  • Physics World Live
  • Impact Explore the value of scientific research for industry, the economy and society
  • Events Plan the meetings and conferences you want to attend with our comprehensive events calendar
  • Innovation showcases A round-up of the latest innovation from our corporate partners
  • Collections Explore special collections that bring together our best content on trending topics
  • Artificial intelligence Explore the ways in which today’s world relies on AI, and ponder how this technology might shape the world of tomorrow
  • #BlackInPhysics Celebrating Black physicists and revealing a more complete picture of what a physicist looks like
  • Nanotechnology in action The challenges and opportunities of turning advances in nanotechnology into commercial products
  • The Nobel Prize for Physics Explore the work of recent Nobel laureates, find out what happens behind the scenes, and discover some who were overlooked for the prize
  • Revolutions in computing Find out how scientists are exploiting digital technologies to understand online behaviour and drive research progress
  • The science and business of space Explore the latest trends and opportunities associated with designing, building, launching and exploiting space-based technologies
  • Supercool physics Experiments that probe the exotic behaviour of matter at ultralow temperatures depend on the latest cryogenics technology
  • Women in physics Celebrating women in physics and their contributions to the field
  • IOP Publishing
  • Enter e-mail address
  • Show Enter password
  • Remember me Forgot your password?
  • Access more than 20 years of online content
  • Manage which e-mail newsletters you want to receive
  • Read about the big breakthroughs and innovations across 13 scientific topics
  • Explore the key issues and trends within the global scientific community
  • Choose which e-mail newsletters you want to receive

Reset your password

Please enter the e-mail address you used to register to reset your password

Note: The verification e-mail to change your password should arrive immediately. However, in some cases it takes longer. Don't forget to check your spam folder.

If you haven't received the e-mail in 24 hours, please contact [email protected]

Registration complete

Thank you for registering with Physics World If you'd like to change your details at any time, please visit My account

  • Quantum mechanics
  • Opinion and reviews

The double-slit experiment

This article is an extended version of the article “The double-slit experiment” that appeared in the September 2002 issue of Physics World (p15). It has been further extended to include three letters about the history of the double-slit experiment with single electrons that were published in the May 2003 issue of the magazine.

Single-electron interference at Bologna

What is the most beautiful experiment in physics? This is the question that Robert Crease asked Physics World readers in May – and more than 200 replied with suggestions as diverse as Schrödinger’s cat and the Trinity nuclear test in 1945. The top five included classic experiments by Galileo, Millikan, Newton and Thomas Young. But uniquely among the top 10, the most beautiful experiment in physics – Young’s double-slit experiment applied to the interference of single electrons – does not have a name associated with it.

Most discussions of double-slit experiments with particles refer to Feynman’s quote in his lectures: “We choose to examine a phenomenon which is impossible, absolutely impossible, to explain in any classical way, and which has in it the heart of quantum mechanics. In reality, it contains the only mystery.” Feynman went on to add: “We should say right away that you should not try to set up this experiment. This experiment has never been done in just this way. The trouble is that the apparatus would have to be made on an impossibly small scale to show the effects we are interested in. We are doing a “thought experiment”, which we have chosen because it is easy to think about. We know the results that would be obtained because there are many experiments that have been done, in which the scale and the proportions have been chosen to show the effects we shall describe”.

It is not clear that Feynman was aware that the first double-slit experiment with electrons had been carried out in 1961, the year he started his lectures (which were published in 1963). More surprisingly, perhaps, Feynman did not stress that an interference pattern would build up even if there was just one electron in the apparatus at a time. (This lack of emphasis was unusual because in the same lecture Feynman describes the electron experiment – and other double-slit experiments with water waves and bullets – in considerable detail).

So who actually carried out the first double-slit experiment with single electrons? Not surprisingly many thought or gedanken experiments are named after theorists – such as the Aharonov-Bohm effect, Bell’s inequality, the Casimir force, the Einstein-Podolsky-Rosen paradox, Schrödinger’s cat and so on – and these names rightly remain even when the experiment has been performed by others in the laboratory. However, it seems remarkable that no name whatsoever is attached to the double-slit experiment with electrons. Standard reference books are silent on this question but a study of the literature reveals several unsung experimental heroes.

Back to Young

Young carried out his original double-slit experiment with light some time in the first decade of the 1800s, showing that the waves of light from the two slits interfered to produce a characteristic fringe pattern on a screen. In 1909 Geoffrey Ingram (G I) Taylor conducted an experiment in which he showed that even the feeblest light source – equivalent to “a candle burning at a distance slightly exceeding a mile” – could lead to interference fringes. This led to Dirac’s famous statement that “each photon then interferes only with itself”.

In 1927 Clinton Davisson and Lester Germer observed the diffraction of electron beams from a nickel crystal – demonstrating the wave-like properties of particles for the first time – and George (G P) Thompson did the same with thin films of celluloid and other materials shortly afterwards. Davisson and Thomson shared the 1937 Nobel prize for “discovery of the interference phenomena arising when crystals are exposed to electronic beams”, but neither performed a double-slit experiment with electrons.

In the early 1950s Ladislaus Laszlo Marton of the US National Bureau of Standards (now NIST) in Washington, DC demonstrated electron interference but this was in a Mach-Zehnder rather than a double-slit geometry. These were the early days of the electron microscope and physicists were keen to exploit the very short de Broglie wavelength of electrons to study objects that were too small to be studied with visible light. Doing gedanken or thought experiments in the laboratory was further down their list of priorities.

A few years later Gottfried Möllenstedt and Heinrich Düker of the University of Tübingen in Germany used an electron biprism – essentially a very thin conducting wire at right angles to the beam – to split an electron beam into two components and observe interference between them. (Möllenstedt made the wires by coating fibres from spiders’ webs with gold – indeed, it is said that he kept spiders in the laboratory for this purpose). The electron biprism was to become widely used in the development of electron holography and also in other experiments, including the first measurement of the Aharonov-Bohm effect by Bob Chambers at Bristol University in the UK in 1960.

But in 1961 Claus Jönsson of Tübingen, who had been one of Möllenstedt’s students, finally performed an actual double-slit experiment with electrons for the first time ( Zeitschrift für Physik 161 454). Indeed, he demonstrated interference with up to five slits. The next milestone – an experiment in which there was just one electron in the apparatus at any one time – was reached by Akira Tonomura and co-workers at Hitachi in 1989 when they observed the build up of the fringe pattern with a very weak electron source and an electron biprism ( American Journal of Physics 57 117-120). Whereas Jönsson’s experiment was analogous to Young’s original experiment, Tonomura’s was similar to G I Taylor’s. (Note added on 7 May: Pier Giorgio Merli, Giulio Pozzi and GianFranco Missiroli carried out double-slit interference experiments with single electrons in Bologna in the 1970s; see Merli et al. in Further reading and the letters from Steeds, Merli et al. , and Tonomura at the end of this article.)

Since then particle interference has been demonstrated with neutrons, atoms and molecules as large as carbon-60 and carbon-70. And earlier this year another famous experiment in optics – the Hanbury Brown and Twiss experiment – was performed with electrons for the first time (again at Tübingen!). However, the results are profoundly different this time because electrons are fermions – and therefore obey the Pauli exclusion principle – whereas photons are bosons and do not.

Credit where it’s due

So why are Jönsson, Tonomura and the other pioneers of the double-slit experiment not well known? One obvious reason is that Jönsson’s results were first published in German in a German journal. Another reason might be that there was little incentive to perform the ultimate thought experiment in the lab, and little recognition for doing so. When Jönsson’s paper was translated into English 13 years later and published in the American Journal of Physics in 1974 (volume 42, pp4-11), the journal’s editors, Anthony (A P) French and Edwin Taylor, described it as a “great experiment”, but added that there are “few professional rewards” for performing what they describe as “real, pedagogically clean fundamental experiments.”

It is worth noting that the first double-slit experiment with single electrons by Tonomura and co-workers was also published in the American Journal of Physics , which publishes articles on the educational and cultural aspects of physics, rather than being a research journal. Indeed, the journal’s information for contributors states: “We particularly encourage manuscripts on already published contemporary research that can be used directly or indirectly in the classroom. We specifically do not publish articles announcing new theories or experimental results.”

French and Taylor’s editorial also confirms how little known Jönsson’s experiment was at the time: “For decades two-slit electron interference has been presented as a thought experiment whose predicted results are justified by their remote and somewhat obscure relation to real experiments in which electrons are diffracted by crystals. Few such recent presentations acknowledge that the two-slit electron interference experiment has now been done and that the results agree with the expectation of quantum physics in all detail.”

However, it should be noted that the history of physics is complicated and that events are rarely as clear-cut as we might like. For instance, it is widely claimed that Young performed his double-slit experiment in 1801 but he did not publish any account of it until his Lectures on Natural Philosophy in 1807. It also appears as if Davisson and a young collaborator called Charles Kunsman observed electron diffraction in 1923 – four years before Davisson and Germer – without realising it.

Final thoughts

Gedanken or thought experiments have played an important role in the history of quantum physics. It is unlikely that the whole area of quantum information would be as lively as it is today – both theoretically and experimentally – if a small band of physicists had not persevered and actually demonstrated quantum phenomena with individual particles.

At one time the Casimir force, which has yet to be measured with an accuracy of better than 15% in the geometry first proposed by Hendrik Casimir in 1948, might also have been viewed as purely a pedagogical experiment – a gedanken experiment with little relevance to real experimental physics. However, it is now clear that applications as varied as nanotechnology and experimental tests of theories of “large” extra dimensions require a detailed knowledge of the Casimir force .

The need for “real, pedagogically clean fundamental experiments” is clearly as great as ever.

This is a longer version of the article “The double-slit experiment” that appeared in the print version of the September issue of Physics World, on page 15. Three letters that appeared in the May 2003 issue of the magazine have been added to the end of this version of the article.

T Young 1802 On the theory of light and colours (The 1801 Bakerian Lecture) Philosophical Transactions of the Royal Society of London 92 12-48

T Young 1804 Experiments and calculations relative to physical optics (The 1803 Bakerian Lecture) Philosophical Transactions of the Royal Society of London 94 1-16

T Young 1807 A Course of Lectures on Natural Philosophy and the Mechanical Arts (J Johnson, London)

G I Taylor 1909 Interference fringes with feeble light Proceedings of the Cambridge Philosophical Society 15 114-115

P A M Dirac 1958 The Principles of Quantum Mechanics (Oxford University Press) 4th edn p9

R P Feynman, R B Leighton and M Sands 1963 The Feynman Lecture on Physics (Addison-Wesley) vol 3 ch 37 (Quantum behaviour)

A Howie and J E Fowcs Williams (eds) 2002 Interference: 200 years after Thomas Young’s discoveries Philosophical Transactions of the Royal Society of London 360 803-1069

R P Crease 2002 The most beautiful experiment Physics World September pp19-20. This article contains the results of Crease’s survey for Physics World ; the first article about the survey appeared on page 17 of the May 2002 issue.

Electron interference experiments

Visit www.nobel.se/physics/laureates/1937/index.html for details of the Nobel prize awarded to Clinton Davisson and George Thomson

L Marton 1952 Electron interferometer Physical Review 85 1057-1058

L Marton, J Arol Simpson and J A Suddeth 1953 Electron beam interferometer Physical Review 90 490-491

L Marton, J Arol Simpson and J A Suddeth 1954 An electron interferometer Reviews of Scientific Instruments 25 1099-1104

G Möllenstedt and H Düker 1955 Naturwissenschaften 42 41

G Möllenstedt and H Düker 1956 Zeitschrift für Physik 145 377-397

G Möllenstedt and C Jönsson 1959 Zeitschrift für Physik 155 472-474

R G Chambers 1960 Shift of an electron interference pattern by enclosed magnetic flux Physical Review Letters 5 3-5

C Jönsson 1961 Zeitschrift für Physik 161 454-474

C Jönsson 1974 Electron diffraction at multiple slits American Journal of Physics 42 4-11

A P French and E F Taylor 1974 The pedagogically clean, fundamental experiment American Journal of Physics 42 3

P G Merli, G F Missiroli and G Pozzi 1976 On the statistical aspect of electron interference phenomena American Journal of Physics 44 306-7

A Tonomura, J Endo, T Matsuda, T Kawasaki and H Ezawa 1989 Demonstration of single-electron build-up of an interference pattern American Journal of Physics 57 117-120

H Kiesel, A Renz and F Hasselbach 2002 Observation of Hanbury Brown-Twiss anticorrelations for free electrons Nature 418 392-394

Atoms and molecules

O Carnal and J Mlynek 1991 Young’s double-slit experiment with atoms: a simple atom interferometer Physical Review Letters 66 2689-2692

D W Keith, C R Ekstrom, Q A Turchette and D E Pritchard 1991 An interferometer for atoms Physical Review Letters 66 2693-2696

M W Noel and C R Stroud Jr 1995 Young’s double-slit interferometry within an atom Physical Review Letters 75 1252-1255

M Arndt, O Nairz, J Vos-Andreae, C Keller, G van der Zouw and A Zeilinger 1999 Wave-particle duality of C 60 molecules Nature 401 680-682

B Brezger, L Hackermüller, S Uttenthaler, J Petschinka, M Arndt and A Zeilinger 2002 Matter-wave interferometer for large molecules Physical Review Letters 88 100404

Review articles and books

G F Missiroli, G Pozzi and U Valdrè 1981 Electron interferometry and interference electron microscopy Journal of Physics E 14 649-671. This review covers early work on electron interferometry by groups in Bologna, Toulouse, Tübingen and elsewhere.

A Zeilinger, R Gähler, C G Shull, W Treimer and W Mampe 1988 Single- and double-slit diffraction of neutrons Reviews of Modern Physics 60 1067-1073

A Tonomura 1993 Electron Holography (Springer-Verlag, Berlin/New York)

H Rauch and S A Werner 2000 Neutron Interferometry: Lessons in Experimental Quantum Mechanics (Oxford Science Publications)

The double-slit experiment with single electrons

The article “A brief history of the double-slit experiment” (September 2002 p15; correction October p17) describes how Claus Jönsson of the University of Tübingen performed the first double-slit interference experiment with electrons in 1961. It then goes on to say: “The next milestone – an experiment in which there was just one electron in the apparatus at any one time – was reached by Akira Tonomura and co-workers at Hitachi in 1989 when they observed the build up of the fringe pattern with a very weak electron source and an electron biprism ( Am. J. Phys. 57 117-120)”.

In fact, I believe that “the first double-slit experiment with single electrons” was performed by Pier Giorgio Merli, GianFranco Missiroli and Giulio Pozzi in Bologna in 1974 – some 15 years before the Hitachi experiment. Moreover, the Bologna experiment was performed under very difficult experimental conditions: the intrinsic coherence of the thermionic electron source used by the Bologna group was considerably lower than that of the field-emission source used in the Hitachi experiment.

The Bologna experiment is reported in a film called “Electron Interference” that received the award in the physics category at the International Festival on Scientific Cinematography in Brussels in 1976. A selection of six frames from the film ( see figure ) was also used for a short paper, “On the statistical aspect of electron interference phenomena”, that was submitted for publication in May 1974 and published two years later (P G Merli, G F Missiroli and G Pozzi 1976 Am. J. Phys. 44 306-7).

John Steeds Department of Physics, University of Bristol [email protected]

The history of science is not restricted to the achievements of big scientists or big scientific institutions. Contributions can also be made by researchers with the necessary background, curiosity and enthusiasm. In the period 1973-1974 we were investigating practical applications of electron interferometry with a Siemens Elmiskop 101 electron microscope that had been carefully calibrated at the CNR-LAMEL laboratory in Bologna, where one of us (PGM) was based ( J. Phys. E7 729-32).

These experiments followed earlier work at the Istituto di Fisica in 1972-73 in which the electron biprism was inserted in a Siemens Elmiskop IA and then used both for didactic ( Am. J. Phys. 41 639-644) and research experiments ( J. Microscopie 18 103-108). We used the Elmiskop 101 for many experiments including, for instance, the observation of the electrostatic field associated with p-n junctions ( J. Microscopie 21 11-20).

During this period we learnt that Professors Angelo and Aurelio Bairati in the Institute of Anatomy at the University of Milan had bought an image intensifier that could be used with the Elmiskop 101. Out of curiosity, and also realizing the conceptual importance of interference experiments with single photons or electrons, we asked if we could attempt to perform an interference experiment with single electrons in the Milan laboratory. Our results formed the basis of the film “Electron interference” and were also published in 1976 ( Am. J. Phys. 44 306-7).

Following the publication of the paper by Tonomura and co-workers in 1989, which did not refer to our 1976 paper (although it did contain an incorrect reference to our film), the American Journal of Physics published a letter from Greyson Gilson of Submicron Structures Inc. The letter stated: “Tonomura et al. seem to believe that they were the first to perform a successful two-slit interference experiment using electrons and also that they were the first to observe the cumulative build-up of the resulting electron interference pattern. Although their demonstration is very admirable, reports of similar work have appeared in this Journal for about 30 years (see, for examples, Refs. 2-7.) It seems inappropriate to permit the widespread misconception that such experiments have not been performed and perhaps cannot be performed to continue.” (G Gilson 1989 Am. J. Phys. 57 680). Three of the seven papers that Gilson refers to were from our group in Bologna.

The main subject of our 1976 paper and the 1989 paper from the Hitachi group are the same: the single-electron build-up of the interference pattern and the statistical aspect of the phenomena. Obviously the electron-detection system used by the Hitachi group in 1989 was more sophisticated than the one we used in 1974. However, the sentence on page 118 of the paper by Tonomura et al. , which states that in our film we “showed the electron arrival in each frame without recording the cumulative arrivals”, is not correct: this can be seen by watching the film and looking at figure 1 of our 1976 paper (a version of which is shown here ).

Finally, it is also worth noting that the first double-slit experiment with single electrons was actually a by-product of research into the practical applications of electron interferometry.

Pier Giorgio Merli LAMEL, CNR Bologna, Italy [email protected] Giulio Pozzi Department of Physics, University of Bologna [email protected] GianFranco Missiroli Department of Physics, University of Bologna [email protected]

The Bologna group photographed the monitor of a sensitive TV camera as they changed the intensity of an electron beam. They observed that a few light flashes of electrons appeared at low intensities, and that interference fringes were formed at high intensities. They also mentioned that they were able to increase the storage time up to “values of minutes”. Historically, they are the first to report such experiments concerning the formation of interference patterns as far as I know.

Later, similar experiments were conducted by Hannes Lichte, then at Tübingen and now at Dresden. Important experiments on electron interference were also carried out by Valentin Fabrikant and co-workers at the Moscow Institute for Energetics in 1949 and later by Takeo Ichinokawa of Waseda University in Tokyo.

Our experiments at Hitachi (A Tonomura, J Endo, T Matsuda, T Kawasaki and H Ezawa 1989 Demonstration of single-electron buildup of an interference pattern Am. J. Phys. 57 117–120) differed from these experiments in the following respects:

(a) Our experiments were carried out from beginning to end with constant and extremely low electron intensities – fewer than 1000 electrons per second – so there was no chance of finding two or more electrons in the apparatus at the same time. This removed any possibility that the fringes might be due to interactions between the electrons, as had been suspected by some physicists, such as Sin-Itiro Tomonaga.

(b) We developed a position-sensitive electron-counting system that was modified from the photon-counting image acquisition system produced by Hamamatsu Photonics. In this system, the formation of fringes could be observed as a time series; the electrons were accumulated over time to gradually form an interference pattern on the monitor (similar to a long exposure with a photographic film). The electrons arrived at random positions on the detector only once in a while and it took more than 20 minutes for the interference pattern to form (see figure). To film the build-up process, the electron source, the electron biprism and the rest of the experiment therefore had to be extremely stable: if the interference pattern had drifted by a fraction of fringe spacing over the exposure time, the whole fringe pattern would have disappeared.

Single-electron interference at Hitachi

(c) The electrons arriving at the detector were detected with almost 100% efficiency. Counting losses and noise in conventional TV cameras mean that it is difficult to know if each flash of the screen really corresponds to an individual electron. Therefore, the detection error in our experiment was limited to less than 1%.

We believe that we carried out the first experiment in which the build-up process of an interference pattern from single-electron events could be seen in real time as in Feynman’s famous double-slit Gedanken experiment under the condition, we emphasize, that there was no chance of finding two or more electrons in the apparatus.

Akira Tonomura Hitachi Advanced Research Laboratory, Saitama, Japan [email protected]

Want to read more?

Note: The verification e-mail to complete your account registration should arrive immediately. However, in some cases it takes longer. Don't forget to check your spam folder.

If you haven't received the e-mail in 24 hours, please contact [email protected] .

  • E-mail Address

Peter Rodgers is editor of Physics World

double slit experiment quora

Physics World Careers

Providing valuable careers advice and a comprehensive employer directory

  • Culture, history and society

Opening the book of revelations

  • Philosophy, sociology and religion

Manufacturing firsts in physics

Discover more from physics world.

Abstract representation of quantum entanglement

Paradigm shifts: positivism, realism and the fight against apathy in the quantum revolution

quantum mechanics abstract illustration

It’s official: United Nations declares 2025 the International Year of Quantum Science and Technology

Abdus Salam list

Abdus Salam: celebrating a unifying force in global physics

Related jobs, application scientist quantum technologies, assistant professor in materials systems for sustainability, master in quantum science & technology, related events.

  • Optics and photonics | Symposium SPIE Optics + Photonics 2024 18—24 August 2024 | San Diego, US
  • Quantum | Workshop III Workshop on Quantum Nonstationary Systems 26—30 August 2024 | Brasilia, Brazil
  • Quantum | Exhibition Quantum Tech Europe 2024 23—25 September 2024 | London, UK

What is the double-slit experiment, and why is it so important?

Reality will surprise you..

John Loeffler

John Loeffler

What is the double-slit experiment, and why is it so important?

Timm Weitkamp/Wikimedia Commons

Few science experiments are as strange and compelling as the double-slit experiment. 

Few experiments, if any, in modern physics are capable of conveying such a simple idea—that light and matter can act as both waves and discrete particles depending on whether they are being observed—but which is nonetheless one of the great mysteries of quantum mechanics.

It’s the kind of experiment that despite its simplicity is difficult to wrap your mind around because what it shows is incredibly counter-intuitive.

But not only has the double-slit experiment been repeated countless times in physics labs around the world, but it has also even spawned many derivative experiments that further reinforce its ultimate result, that particles can be waves or discrete objects and that it is as if they “know” when you are watching them.

What does the double-slit experiment demonstrate?

To understand what the double-slit experiment demonstrates, we need to lay out some key ideas from quantum mechanics.

In 1925, Werner Heisenberg presented his mentor, the eminent German physicist Max Born, with a paper to review that showed how the properties of subatomic particles, like position, momentum, and energy, could be measured. 

Born saw that these properties could be represented through mathematical matrices, with definite figures and descriptions of individual particles, and this laid the foundation for the matrix description of quantum mechanics .

Meanwhile, in 1926,  Edwin Schrödinger  published his wave theory of quantum mechanics which showed that particles could be described by an equation that defined their waveform; that is, it determined that particles were actually waves.

This gave rise to the concept of wave-particle duality, which is one of the defining features of quantum mechanics. According to this concept, subatomic entities can be described as both waves and particles, and it is up to the observer to decide how to measure them.

That last part is important since it will determine how quantum entities will manifest. If you try to measure a particle’s position, you will measure a particle’s position, and it will cease to be a wave at all.

If you try to define its momentum, you will find that behaves like a wave and you can’t know anything definitive about its position beyond the probability that it exists at any given point within that wave.

Essentially, you will measure it as a particle or a wave, and doing so decides what form it will take.

The double-slit experiment is one of the simplest demonstrations of this wave-particle duality as well as a central defining weirdness of quantum mechanics, one that makes the observer an active participant in the fundamental behavior of particles.

How does the double-slit experiment work?

The easiest way to describe the double-slit experiment is by using light. First, take a source of coherent light, such as a laser beam, that shines in a single wavelength, like purely blue visible light at 460nm, and aim it at a wall with two slits in it.  The distance between the slits should be roughly the same as the light’s wavelength  so that they will both sit inside that beam of light.

Behind that wall, place a screen that can detect and record the light that impacts it. If you fire the laser beam at the two slits, on the recording screen behind the wall you will see a stripey pattern like this:

double slit experiment quora

This is probably not what you might have been expecting, and that’s perfectly rational if you treat light as if it were a wave. If the light was a wave, then when the single wave of light from the laser hit both slits, each slit would become a new “source” of light on the other side of the wall, and so you would have a new wave originating from each slit producing two waves.

Where those two waves intersect causes something known as interference, and it can be either constructive or destructive. When the amplitude of the waves overlaps at either a peak or a trough, it acts to boost the wavelength in either direction by adding its energy together. This is constructive interference, and it produces these brighter bars in this pattern.

When the waves cancel each other out, as when a peak hits a trough, the effect neutralizes the wavelength and diminishes or even eliminates the light, producing the blacked-out spaces in between the blue bars.

But in the case of quantum entities like photons of light or electrons, they are also individual particles. So what happens when you shoot a single photon through the double slits?

One photon alone reacting to the screen might leave a tiny dot behind, which might not mean much in isolation, but if you shoot many single photons at the double slits, those tiny dots that the photon leaves behind on our screen actually show up in that same stripey interference pattern produced by the laser beam hitting the double slits.

In other words, the individual photon behaves as if it passed through both slits like it was a wave.

Now, here’s where things get really weird.

We can set up a detector in front of one of the slits that can watch for photons and light up whenever it detects one passing through. When we do this, the detector will light up 50% of the time, and the pattern left behind on the screen changes, giving us something that looks like this:

double slit experiment quora

And to make things even wilder, we can set up a detector behind the wall that only detects a photon after it has passed through the slit and we get the same result. That means that even if the photon passes through both slits as a wave, the moment it is detected, it is no longer a wave but a particle. And not just that, that second wave emerging from the other slit also collapses back into the particle that was detected passing through the other slit.

In practice, this means that somehow the universe “knows” that someone is watching and flips the metaphorical quantum coin to see which slit the particle passed through. The more individual photons you shoot through the double slit, the closer that photon detector comes to detecting photons 50% of the time, just as flipping a coin 10 times might give you heads 70% of the time while flipping it 100 times might give you tails 55% of the time, and flipping it 1 billion times gives you heads 50.0003% of the time.

This seems to show that not only is the universe watching the observer as well, but that the quantum states of entities passing through the double slits are governed by the laws of probability, making it impossible to ever predict with certainty what the quantum state of an entity will be.

Who invented the double-slit experiment?

The double-slit experiment actually predates quantum mechanics by a little more than a century.

During the Scientific Revolution, the nature of light was a particularly contentious topic, with many—like Isaac Newton himself—arguing in favor of a corpuscular theory of light that held that light was transmitted through particles.

Others believed that light was a wave that was transmitted through “aether” or some other medium, the way sound travels through air and water, but Newton’s reputation and a lack of an effective means to demonstrate the wave theory of light solidified the corpuscular view for just shy of a century after Newton published his  Opticks in 1704 .

The definitive demonstration came from the British polymath Thomas Young, who presented a paper to the Royal Society of London in 1803 that described a pair of simple experiments that anyone could perform to see for themselves that light was in fact a wave.

First, Young established that a pair of waves were subject to interference when they overlapped, producing a distinctive interference pattern.

double slit experiment quora

He initially demonstrated this interference pattern using a ripple tank of water, showing that such a pattern is characteristic of wave propagation.

Young then introduced the precursor to the modern double-slit experiment, though instead of using a laser beam to produce the required light source, Young used reflected sunlight striking two slits in a card as its target.

The resulting light diffraction showed the expected interference pattern, and the wave theory of light gained considerable support. It would take another decade and a half before further experimentation conclusively refuted corpuscles in favor of waves, but the double-slit experiment that Young developed proved to be a fatal blow to Newton’s theory.

How to do the double-slit experiment

Young wasn’t lying when he said , “The experiments I am about to relate…may be repeated with great ease, whenever the sun shines, and without any other apparatus than is at hand to everyone.”

While it might be a stretch to say that you can use the double-slit experiment to demonstrate some of the more counterintuitive features of quantum mechanics (unless you have a photon detector handy and a laser that shoots individual photons), you can still use it to demonstrate the wave nature of light.

If you want to replicate Young’s experiment, you only need as large a box as is practical with a hole cut in it a little smaller than an index card. Then, take an Exacto knife or similar blade for fine cutting work and cut two slits into a piece of cardboard larger than the hole in your box. The slits should be between 0.1mm and 0.4mm apart, as the closer together they are, the more distinct the interference pattern will be. It’s better to create cards for this rather than cut directly into the box since you might need to make adjustments to the spacing of the slits.

Once you’re satisfied with the spacing, affix the card with the double-slit in it over the hole and secure it in place with tape. Just make sure sunlight isn’t leaking around the card.

You’ll also need to create some eye-holes in the box so you can look inside without getting in the way of the light hitting the double-slit card, but once you figure that out, you’re all set.

To accurately diffract sunlight using this box, you will need to have the sunlight more or less hitting the double-slit card dead on, so it might take some maneuvering to get it properly positioned.

Once it is, look through the eye holes and you can see the interference pattern forming on the inside wall, as well as different colors emerging as the different wavelengths interfering with each other change the color of the light being created.

If you wanted to try it out with something fancier, get yourself a laser pointer from an office supply store. Just like you’d do with a viewing box, create cards with slits in them, and when properly spaced, set up a shielded area for the card to rest on.

You’ll want to make sure that only the light from the laser pointer is hitting the double-slit, so shield the card however you need to. Then, set the laser pointer on a surface level with the slits and shine the laser at them. On the wall behind the card, the interference pattern from the slits should be clearly visible.

If you don’t want to go through all that trouble, you can also use Photoshop or similar software to recreate the effect.

First, create a template of evenly spaced concentric circles. Using different layers for each source, as well as a background later, position the center of the concentric rings near to one another. On a 1200 pixel wide canvas, a distance of 100 pixels between the two centers should do nicely.

Then, fill in the color of each concentric ring, alternating light and dark, with an opacity set to about 33%. You may need to hide one of the concentric circle layers while you work on the other. When you’re done, reveal the two overlapping layers of circles and the interference pattern should jump out at you immediately, looking something like this:

RECOMMENDED ARTICLES

double slit experiment quora

Of course, if you want to dig into the quantum mechanics side of things, you’ll need to work in a pretty advanced physics lab at a university or science institute, since photon detectors aren’t the kind of thing you can pick up at the hobby store.

Still, if you’re compelled to try the heavier stuff out for yourself, you wouldn’t be the first person to get drawn into a career in physics because of the weirdness of quantum mechanics, and there are definitely worse ways to make a living.

The Blueprint Daily

Stay up-to-date on engineering, tech, space, and science news with The Blueprint.

By clicking sign up, you confirm that you accept this site's Terms of Use and Privacy Policy

ABOUT THE EDITOR

John Loeffler John is a writer and programmer living in New York City. He writes about computers, gadgetry, gaming, VR/AR, and related consumer technologies. You can find him on Twitter @thisdotjohn

POPULAR ARTICLES

Your microwave harbors radiation-resistant bacteria similar to solar panels, 2,006 meters a day: chinese oil exploration sets new drilling record, chapelcross nuclear site to be turned into hydrogen production, green energy hub, uss pierre: austal launches us navy’s newest independence-variant combat ship, related articles.

Nanoparticles could warm Mars by 50°Fahrenheit, make planet habitable

Nanoparticles could warm Mars by 50°Fahrenheit, make planet habitable

World’s 1st 4th gen heat pump to boost EV performance, driving range

  • transportation

World’s 1st 4th gen heat pump to boost EV performance, driving range

New lensless fluorescence microbial detection system to save millions of lives

New lensless fluorescence microbial detection system to save millions of lives

1,800 bhp beast: Rolls-Royce’s WW2 legacy aero engine to be auctioned

1,800 bhp beast: Rolls-Royce’s WW2 legacy aero engine to be auctioned

Thank you for visiting nature.com. You are using a browser version with limited support for CSS. To obtain the best experience, we recommend you use a more up to date browser (or turn off compatibility mode in Internet Explorer). In the meantime, to ensure continued support, we are displaying the site without styles and JavaScript.

  • View all journals
  • Explore content
  • About the journal
  • Publish with us
  • Sign up for alerts
  • BOOKS AND ARTS
  • 07 August 2018

Two slits and one hell of a quantum conundrum

  • Philip Ball 0

Philip Ball is a writer based in London.

You can also search for this author in PubMed   Google Scholar

You have full access to this article via your institution.

Blue light splitting into wave like interference pattern during a double slit experiment.

Bands of light in a double-slit experiment. Credit: Timm Weitkamp/CC BY 3.0

Through Two Doors at Once: The Elegant Experiment That Captures the Enigma of Our Quantum Reality Anil Ananthaswamy Dutton (2018)

According to the eminent physicist Richard Feynman, the quantum double-slit experiment puts us “up against the paradoxes and mysteries and peculiarities of nature”. By Feynman’s logic, if we could understand what is going on in this deceptively simple experiment, we would penetrate to the heart of quantum theory — and perhaps all its puzzles would dissolve.

That’s the premise of Through Two Doors at Once . Science writer Anil Ananthaswamy focuses on this single experiment, which has taken many forms since quantum mechanics debuted in the early twentieth century with the work of Max Planck, Albert Einstein, Niels Bohr and others. In some versions, nature seems magically to discern our intentions before we enact them — or perhaps retroactively to alter the past. In others, the outcome seems dependent on what we know, not what we do. In yet others, we can deduce something about a system without looking at it. All in all, the double-slit experiment seems, to borrow from Feynman again, “screwy”.

The original experiment, as Ananthaswamy notes, was classical, conducted by British polymath Thomas Young in the early 1800s to show that light is a wave. He passed light through two closely spaced parallel slits in a screen, and on the far side saw several bright bands. This, he realized, was an ‘interference’ pattern. Caused by the interaction of waves emanating from the openings, it’s not unlike the pattern that appears when two pebbles are dropped into water and the ripples they create add to or dampen each other’s peaks and troughs. With ordinary particles, the slits would act more like stencils for sprayed paint, creating two defined bands.

We now know that quantum particles create such an interference pattern, too — evidence that they have a wave-like nature. Postulated in 1924 by French physicist Louis de Broglie, this idea was verified for electrons a few years later by US physicists Clinton Davisson and Lester Germer. Even large molecules such as buckminsterfullerene — made of 60 carbon atoms — will behave in this way.

You can get used to that. What’s odd is that the interference pattern remains — accumulating over many particle impacts — even if particles go through the slits one at a time. The particles seem to interfere with themselves. Odder, the pattern vanishes if we use a detector to measure which slit the particle goes through: it’s truly particle-like, with no more waviness. Oddest of all, that remains true if we delay the measurement until after the particle has traversed the slits (but before it hits the screen). And if we make the measurement but then delete the result without looking at it, interference returns.

It’s not the physical act of measurement that seems to make the difference, but the “act of noticing”, as physicist Carl von Weizsäcker (who worked closely with quantum pioneer Werner Heisenberg) put it in 1941. Ananthaswamy explains that this is what is so strange about quantum mechanics: it can seem impossible to eliminate a decisive role for our conscious intervention in the outcome of experiments. That fact drove physicist Eugene Wigner to suppose at one point that the mind itself causes the ‘collapse’ that turns a wave into a particle.

Ananthaswamy offers some of the most lucid explanations I’ve seen of other interpretations. Bohr’s answer was that quantum mechanics doesn’t let us say anything about the particle’s ‘path’ — one slit or two — before it is measured. The role of the theory, said Bohr, is to furnish predictions of measurement outcomes; in that regard, it has never been found to fail. (However, he did not, as is often implied, deny that there is any physical reality beyond measurement.) Yet this does feel rather unsatisfactory. Ananthaswamy seems tempted by the alternative idea offered by David Bohm in the 1950s. Here, quantum objects are both particle and wave, the wave somehow ‘piloting’ the particle through space while being sensitive to influences beyond the particle’s location. But Ananthaswamy concludes that “physics has yet to complete its passage through the double-slit experiment. The case remains unsolved.”

With apologies to researchers convinced that they have the answer, this is true: there is no consensus. At any rate, Bohr was right to advise caution in how we use language. There is nothing in quantum mechanics as it stands, shorn of interpretation, that lets us speak of particles becoming waves or taking two paths at once. And there is no reason to regard the wavefunction as more or less than an abstraction. This mathematical function, which embodies all we can know about a quantum object (and features in the iconic equation devised by Erwin Schrödinger to describe the object’s wave-like behaviour) was characterized rather nicely by physicist Roland Omnès. He called it “the fuel of a machine that manufactures probabilities” — that is, probabilities of measurement outcomes.

Refracting all of quantum mechanics through the double slits is both a strength and a weakness of Through Two Doors at Once . It brings unity to a knotty subject, but downplays some important strands. Those include John Bell’s 1964 thought experiment on the nature of quantum entanglement (conducted for real many times since the 1970s); the role of decoherence in the emergence of classical physics from quantum phenomena (adduced in the 1970s and 1980s); and the emphasis on information and causality in the past two decades. Still, given that popularization of quantum mechanics seems to be the flavour of the month — summoning Adam Becker’s 2018 book What is Real? , Jean Bricmont’s 2017 Quantum Sense and Nonsense , a forthcoming book by physicist Sean Carroll, and my own 2018 Beyond Weird — there’s no lack of a wider perspective.

And we need that. Ananthaswamy’s conclusion — that perhaps all the major interpretations are “touching the truth in their own way” — is not a shrugging capitulation. It’s a well-advised commitment to pluralism, shared with Becker’s book and mine. For now, uncertainty seems the wisest position in the quantum world.

Nature 560 , 165 (2018)

doi: https://doi.org/10.1038/d41586-018-05892-6

Related Articles

double slit experiment quora

  • Quantum physics

Quantum computing aims for diversity, one qubit at a time

Quantum computing aims for diversity, one qubit at a time

Technology Feature 05 AUG 24

The quantum transition of the two-dimensional Ising spin glass

The quantum transition of the two-dimensional Ising spin glass

Article 10 JUL 24

Antiferromagnetic phase transition in a 3D fermionic Hubbard model

Antiferromagnetic phase transition in a 3D fermionic Hubbard model

From Vikings to Beethoven: what your DNA says about your ancient relatives

From Vikings to Beethoven: what your DNA says about your ancient relatives

News Feature 07 AUG 24

How to help students enjoy physics lessons

How to help students enjoy physics lessons

News & Views 06 AUG 24

Maxine Singer obituary: biologist who shaped genetic engineering and fought discrimination

Maxine Singer obituary: biologist who shaped genetic engineering and fought discrimination

Obituary 02 AUG 24

Laboratory Head in Cellular Therapies

Bring your passion for cancer research to an Institute committed to delivering better health and wellbeing through impactful medical research

Herston, Brisbane (AU)

QIMR Berghofer

double slit experiment quora

Postdoctoral Fellowships: Immuno-Oncology (Please specify the project(s) applying)

We currently have multiple postdoctoral fellowship positions available within our multidisciplinary research teams based In Hong Kong.

Hong Kong (HK)

Centre for Oncology and Immunology

double slit experiment quora

Faculty Positions in School of Engineering, Westlake University

The School of Engineering (SOE) at Westlake University is seeking to fill multiple tenured or tenure-track faculty positions in all ranks.

Hangzhou, Zhejiang, China

Westlake University

double slit experiment quora

Locum Chief Editor, Nature Reviews Methods Primers

Job Title: Locum Chief Editor, Nature Reviews Methods Primers Location(s): New York or Heidelberg (hybrid working) Closing date: August 23rd, 2024 ...

New York City, New York (US)

Springer Nature Ltd

double slit experiment quora

Associate or Senior Editor, BMC Neurology portfolio

Job Title: Associate or Senior Editor, BMC Neurology portfolio Locations: New York or Heidelberg Application Deadline: August 23rd, 2024   About Sp...

double slit experiment quora

Sign up for the Nature Briefing newsletter — what matters in science, free to your inbox daily.

Quick links

  • Explore articles by subject
  • Guide to authors
  • Editorial policies

What Is The Double-Slit Experiment?

The double-slit experiment, observation affects reality, the various interpretations:.

The double-slit experiment shows the duality of the quantum world. A photon’s wave/particle duality is affected when it is observed.

Light has been one of the major areas of inquiry for physicists since we first began questioning the world around us. Understandably so, as it is the medium by which we see, measure and understand the world. It holds a powerful symbolism in our imaginations, is reflected in our religions and is famously quoted in our scriptures.

let there be light meme

Rigorous science has enlightened our ignorance about Light. Until the 1800s, light was thought to be made up of particles, attested by Newtonian physics.

This came rather intuitively, as we see light traveling in a straight line, like bullets coming out of a gun.

Prison cell interior , sunrays coming through a barred window - Illustration(nobeastsofierce)S

However, nature is often weirder than our expectations and light’s weird behavior was first shown by Thomas Young in his now heavily worked upon and immortalized double-slit experiment. This experiment provides some fascinating insights into the minute workings of nature and has challenged everything we know about light, matter, and reality itself. Let’s revisit the experiment that has baffled legendary scientists – including Einstein!

Recommended Video for you:

The experiment is pretty straightforward, with very few parts. There are three main components:

  • A source of light or matter – photons, electrons, bullets

Futuristic Gun Vector Illustrator. Weapon flat icon(VectorKnight)s

• Two narrow slits for the source to pass-through.

Two slits

• A projection screen, where the source makes its impression. The pattern of the impression tells us if it is a wave or a particle.

Projector for impression

The objective of the experiment is to see the underlying make-up of light and matter.

Let’s start with something familiar, bullets from a machine gun. Our gun fires bullets at regular intervals towards the range of the two slits.

The experiment done with bullets

Two straight lines appear on the projector. The graph pattern is that of two mountains; where the crests imply the impression points, and the troughs imply empty places.

The thing to note here is, if we close S2 and fire the gun, only one line appears. Thus, we can safely say that S is equal to the sum of S1 and S2, i.e., S = S1 + S2.

S = S1 + S2

Light is the next source for the experiment. The impression appears as bundles of photons fire through the two slits. What is the pattern you think will emerge? Is it similar to that of bullets? Well, light consists of bullet-like particles, so it isn’t farfetched to say the pattern will be the same.

interference of waves passing a double slit - diffraction pattern - Illustration(magnetix)S

To everyone’s surprise, however, the impression isn’t of two straight lines. The graph pattern that emerges is an interference pattern; there is the brightest impression in the center, and recedes on both sides. An interference pattern is only made when two waves interfere with each other; there is no other possible explanation to it. The pattern shows that the light is moving in waves.

The waves from the two slits are colliding with each other. There is a peculiar formation that develops when two waves collide. A wave has a crest (the part above) and a trough (the part below).

parts of a wave - Vector(Kicky_princess)s

When the crest of one wave collides with the crest of another, it adds and forms constructive interference, seen as a bright spot on the projector. When the crest of one wave and trough of another collide, they cancel each other out and form a destructive pattern, which results in dark spots between the impressions on the projector.

Interference and Phase - Vector(Fouad A. Saad)s

The second slit is closed and the experiment is done again. Now with one slit to move through, the photons form a straight line. Note, here S is not equal to the sum of S1 and S2, and this is also where light’s wave-particle duality comes into focus.

S is not equal to S1 + S2

This revelation changed our thinking about light, but the rabbit hole doesn’t end there; things just get weirder when we further iterate the experiment. Now, instead of flashing a bunch of photons together, only single photons are fired through the slits at regular intervals. Given that it is a single photon, and has no other wave to interact with, we can say that the photon will make a single line on the projector, yet the result is counter-intuitive; the formation on the projector is still an interference pattern!

interference of waves passing a double slit - diffraction pattern - Illustration(magnetix)S

How can this be? How can a single photon know about the photons coming after it and form a pattern resembling that of the group being shot together?

This is where quantum spookiness begins and things get pretty far out. It appears that a single photon is traveling through both the slits and colliding with itself to form the interference pattern. This has bothered physicists a lot, as it does not obey the laws that we see in our Newtonian scale. It turns out that a large assemblage appears to behave in a way that is different from the behavior of its minuscule components.

Now, let’s hit a home run and take this weirdness to another level. This last iteration of the experiment will make you appreciate nature’s absurdness and how totally wacky our world truly is.

Also Read: Wave-Particle Duality: Is An Electron A Particle Or A Wave?

At this point, we have established that a single photon travels from both the slit at the same time and collides with itself to form the interference pattern. As classical physics dictates, it is impossible for the same photon to move through the two slits at the same time. Perhaps it is splitting itself into two parts and interacting with itself. The only way to know is to watch. A detector is placed in one of the slits so when the photon passes through the slit, the detector identifies it.

Adding a detector

As the photon passes through the slit, the detector identifies it. The pattern that emerges on the projector is a single line.

The photon acts as a particle when being observed

Just when you think you’re coming to terms with the quantum scale, things slip over your head. The act of measuring or observing the photon makes it go through only one path, making the impression on the projector of a particle. It doesn’t interact with itself anymore and no interference pattern emerges. When the experiment is carried out with varying degrees of detection, so that the detection is dimmer on every passing photon (say 7-10 photons are being detected and that number keeps decreasing), then the interference pattern starts to slowly emerge again. The photons act as a wave when not being observed and act as particles when they are being observed.

mind blown meme

Also Read: What Is The Observer Effect In Quantum Mechanics?

The double-slit experiments is one of the most iterated experiments in scientific history. Electrons, atoms, molecules and even complex fullerenes like Buckyballs have been used as sources for the experiment. The same results are obtained using every source; the pattern is consistent in both light and matter.

Things on the quantum scale don’t follow the deterministic laws of the macro scale. There are many interpretations of this quantum phenomenon. The Copenhagen Interpretation states that the interference pattern is all the probable functions of the photon (a wave function) and the act of observing or measuring it makes the wave select one of the many alternatives (collapsing of the wave function).

Another interpretation is the many-worlds theory, which states that all the possible states of the photon’s wave function exist simultaneously and our detection is just this particular instance of the wave function.

The theories tend to run wild and it’s safe to say that the quantum realm is a little slippery to wrap your head around. However, there’s no need to feel bad, as you’re in good company. As Richard Feynman said:

Also Read: Why Is Quantum Mechanics So Difficult To Understand?

that nobody understands quantum mechanics meme

  • Copenhagen Interpretation. The University of Oregon
  • A Review and Response to the Book "The Grand Design" by ... - TASC. tasc-creationscience.org
  • Chapter 14 Interference and Diffraction. web.mit.edu
  • Young’s Double Slit Experiment - pressbooks.online.ucf.edu
  • Lecture Notes | Quantum Physics III - MIT OpenCourseWare. MIT OpenCourseWare

Vishal is an Architect and a design aficionado. He likes making trippy patterns in his computer. Fascinated by technology’s role in humanity’s evolution, he is constantly thinking about how the future of our species would turn out – sometimes at the peril of what’s currently going on around him.

What Is Light? Matter Or Energy?

What Is Light? Matter Or Energy?

cd rainbow

What Is Diffraction And Diffraction Grating?

light

Interferometer: What Is The Michelson Interferometer Experiment?

quantum Mechanics

What Is Quantum Mechanics?

harry-potter-g0b8ce416f_1920

Without Magic, Could You Get Through Platform 9 ¾?

Gravitation,Waves,Around,Black,Hole,In,Space,3d,Illustration

What Is Quantum Physics?

double slit experiment quora

Quantum Entanglement: Explained in REALLY SIMPLE Words

double slit experiment quora

Photoelectric Effect Explained in Simple Words for Beginners

double slit experiment quora

What Would Happen If You Traveled At The Speed of Light?

double slit experiment quora

What is the Heisenberg Uncertainty Principle: Explained in Simple Words

double slit experiment quora

Quantum Physics: Here’s Why Movies Always Get It Wrong

double slit experiment quora

Time Dilation - Einstein's Theory Of Relativity Explained!

Logo for British Columbia/Yukon Open Authoring Platform

Want to create or adapt books like this? Learn more about how Pressbooks supports open publishing practices.

3.4 Young’s Double Slit Experiment

  • Explain the phenomena of interference.
  • Define constructive interference for a double slit and destructive interference for a double slit.

Although Christiaan Huygens thought that light was a wave, Isaac Newton did not. Newton felt that there were other explanations for colour, and for the interference and diffraction effects that were observable at the time. Owing to Newton’s tremendous stature, his view generally prevailed. The fact that Huygens’s principle worked was not considered evidence that was direct enough to prove that light is a wave. The acceptance of the wave character of light came many years later when, in 1801, the English physicist and physician Thomas Young (1773–1829) did his now-classic double slit experiment (see Figure 1 ).

A beam of light strikes a wall through which a pair of vertical slits is cut. On the other side of the wall, another wall shows a pattern of equally spaced vertical lines of light that are of the same height as the slit.

Why do we not ordinarily observe wave behaviour for light, such as observed in Young’s double slit experiment? First, light must interact with something small, such as the closely spaced slits used by Young, to show pronounced wave effects. Furthermore, Young first passed light from a single source (the Sun) through a single slit to make the light somewhat coherent. By coherent , we mean waves are in phase or have a definite phase relationship. Incoherent means the waves have random phase relationships. Why did Young then pass the light through a double slit? The answer to this question is that two slits provide two coherent light sources that then interfere constructively or destructively. Young used sunlight, where each wavelength forms its own pattern, making the effect more difficult to see. We illustrate the double slit experiment with monochromatic (single colour, single wavelength,   λ ) light to clarify the effect. Figure 2 shows the pure constructive and destructive interference of two waves having the same wavelength and amplitude.

Figure a shows three sine waves with the same wavelength arranged one above the other. The peaks and troughs of each wave are aligned with those of the other waves. The top two waves are labeled wave one and wave two and the bottom wave is labeled resultant. The amplitude of waves one and two are labeled x and the amplitude of the resultant wave is labeled two x. Figure b shows a similar situation, except that the peaks of wave two now align with the troughs of wave one. The resultant wave is now a straight horizontal line on the x axis; that is, the line y equals zero.

When light passes through narrow slits, it is diffracted into semicircular waves, as shown in Figure 3 (a). Pure constructive interference occurs where the waves are crest to crest or trough to trough. Pure destructive interference occurs where they are crest to trough. The light must fall on a screen and be scattered into our eyes for us to see the pattern. An analogous pattern for water waves is shown in Figure 3 (b). Note that regions of constructive and destructive interference move out from the slits at well-defined angles to the original beam. These angles depend on wavelength and the distance between the slits, as we shall see below.

The figure contains three parts. The first part is a drawing that shows parallel wavefronts approaching a wall from the left. Crests are shown as continuous lines, and troughs are shown as dotted lines. Two light rays pass through small slits in the wall and emerge in a fan-like pattern from two slits. These lines fan out to the right until they hit the right-hand wall. The points where these fan lines hit the right-hand wall are alternately labeled min and max. The min points correspond to lines that connect the overlapping crests and troughs, and the max points correspond to the lines that connect the overlapping crests. The second drawing is a view from above of a pool of water with semicircular wavefronts emanating from two points on the left side of the pool that are arranged one above the other. These semicircular waves overlap with each other and form a pattern much like the pattern formed by the arcs in the first image. The third drawing shows a vertical dotted line, with some dots appearing brighter than other dots. The brightness pattern is symmetric about the midpoint of this line. The dots near the midpoint are the brightest. As you move from the midpoint up, or down, the dots become progressively dimmer until there seems to be a dot missing. If you progress still farther from the midpoint, the dots appear again and get brighter, but are much less bright than the central dots. If you progress still farther from the midpoint, the dots get dimmer again and then disappear again, which is where the dotted line stops.

To understand the double slit interference pattern, we consider how two waves travel from the slits to the screen, as illustrated in Figure 4 . Each slit is a different distance from a given point on the screen. Thus different numbers of wavelengths fit into each path. Waves start out from the slits in phase (crest to crest), but they may end up out of phase (crest to trough) at the screen if the paths differ in length by half a wavelength, interfering destructively as shown in Figure 4 (a). If the paths differ by a whole wavelength, then the waves arrive in phase (crest to crest) at the screen, interfering constructively as shown in Figure 4 (b). More generally, if the paths taken by the two waves differ by any half-integral number of wavelengths ( 1/2λ, 3/2 λ, 5/2 λ …   etc.) then destructive interference occurs. Similarly, if the paths taken by the two waves differ by any integral number of wavelengths (1 λ, 2λ, 3λ…  , etc.), then constructive interference occurs.

Take-Home Experiment: Using Fingers as Slits

Look at a light, such as a street lamp or incandescent bulb, through the narrow gap between two fingers held close together. What type of pattern do you see? How does it change when you allow the fingers to move a little farther apart? Is it more distinct for a monochromatic source, such as the yellow light from a sodium vapour lamp, than for an incandescent bulb?

Both parts of the figure show a schematic of a double slit experiment. Two waves, each of which is emitted from a different slit, propagate from the slits to the screen. In the first schematic, when the waves meet on the screen, one of the waves is at a maximum whereas the other is at a minimum. This schematic is labeled dark (destructive interference). In the second schematic, when the waves meet on the screen, both waves are at a minimum.. This schematic is labeled bright (constructive interference).

Figure 5 below shows how to determine the path length difference for waves traveling from two slits to a common point on a screen. If the screen is a large distance away compared with the distance between the slits, then the angle  θ between the path and a line from the slits to the screen (see the figure) is nearly the same for each path. The difference between the paths is shown in the figure; simple trigonometry shows it to be dsinθ, where  d is the distance between the slits. To obtain constructive interference for a double slit , the path length difference must be an integral multiple of the wavelength, or

Similarly, to obtain destructive interference for a double slit , the path length difference must be a half-integral multiple of the wavelength, or

where  λ  is the wavelength of the light,  d is the distance between slits, and θ is the angle from the original direction of the beam as discussed above. We call  m the order of the interference. For example,  m=4 is fourth-order interference.

The figure is a schematic of a double slit experiment, with the scale of the slits enlarged to show the detail. The two slits are on the left, and the screen is on the right. The slits are represented by a thick vertical line with two gaps cut through it a distance d apart. Two rays, one from each slit, angle up and to the right at an angle theta above the horizontal. At the screen, these rays are shown to converge at a common point. The ray from the upper slit is labeled l sub one, and the ray from the lower slit is labeled l sub two. At the slits, a right triangle is drawn, with the thick line between the slits forming the hypotenuse. The hypotenuse is labeled d, which is the distance between the slits. A short piece of the ray from the lower slit is labeled delta l and forms the short side of the right triangle. The long side of the right triangle is formed by a line segment that goes downward and to the right from the upper slit to the lower ray. This line segment is perpendicular to the lower ray, and the angle it makes with the hypotenuse is labeled theta. Beneath this triangle is the formula delta l equals d sine theta.

The equations for double slit interference imply that a series of bright and dark lines are formed. For vertical slits, the light spreads out horizontally on either side of the incident beam into a pattern called interference fringes, illustrated in Figure 6 . The intensity of the bright fringes falls off on either side, being brightest at the centre. The closer the slits are, the more is the spreading of the bright fringes. We can see this by examining the equation

For fixed    λ   and m , the smaller  d is, the larger  θ must be, since dsinθ= mλ.

This is consistent with our contention that wave effects are most noticeable when the object the wave encounters (here, slits a distance  d apart) is small. Small  d gives large θ , hence a large effect.

The figure consists of two parts arranged side-by-side. The diagram on the left side shows a double slit arrangement along with a graph of the resultant intensity pattern on a distant screen. The graph is oriented vertically, so that the intensity peaks grow out and to the left from the screen. The maximum intensity peak is at the center of the screen, and some less intense peaks appear on both sides of the center. These peaks become progressively dimmer upon moving away from the center, and are symmetric with respect to the central peak. The distance from the central maximum to the first dimmer peak is labeled y sub one, and the distance from the central maximum to the second dimmer peak is labeled y sub two. The illustration on the right side shows thick bright horizontal bars on a dark background. Each horizontal bar is aligned with one of the intensity peaks from the first figure.

Example 1: Finding a Wavelength from an Interference Pattern

Suppose you pass light from a He-Ne laser through two slits separated by 0.0100 mm and find that the third bright line on a screen is formed at an angle of 10.95 o relative to the incident beam. What is the wavelength of the light?

The third bright line is due to third-order constructive interference, which means that m=3 . We are given d = 0.0100 mm and θ = 10.95 o . The wavelength can thus be found using the equation d sinθ = m λ for constructive interference.

The equation is  d sinθ = m λ.  Solving for the wavelength  λ  gives

Substituting known values yields

λ = (d sinθ) / m  = (0.0100 x 10 -3 m) (sin 10.95 o )  /  3 = 6.33 x 10 -7 m  or 

λ  =  633 nm

To three digits, this is the wavelength of light emitted by the common He-Ne laser. Not by coincidence, this red colour is similar to that emitted by neon lights. More important, however, is the fact that interference patterns can be used to measure wavelength. Young did this for visible wavelengths. This analytical technique is still widely used to measure electromagnetic spectra. For a given order, the angle for constructive interference increases with λ , so that spectra (measurements of intensity versus wavelength) can be obtained.

Example 2: Calculating Highest Order Possible

Interference patterns do not have an infinite number of lines, since there is a limit to how big  m can be. What is the highest-order constructive interference possible with the system described in the preceding example?

Strategy and Concept

The equation  d sinθ = mλ  for  m = 0,  1, -1, 2, -2, 3, -3, 4, -4, …  describes constructive interference. For fixed values of  d and λ , the larger  m is, the larger  sinθ is. However, the maximum value that sinθ can have is 1, for an angle of 90 o . (Larger angles imply that light goes backward and does not reach the screen at all.) Let us find which  m corresponds to this maximum diffraction angle.

Solving the equation  d sinθ = mλ   for  m gives

Taking sinθ =sin90 o = 1 and substituting the values of  d and  λ, and remembering the values for the metric prefixes, from the preceding example gives

Therefore, the largest integer m can be is 15, or

The number of fringes depends on the wavelength and slit separation. The number of fringes will be very large for large slit separations. However, if the slit separation becomes much greater than the wavelength, the intensity of the interference pattern changes so that the screen has two bright lines cast by the slits, as expected when light behaves like a ray. We also note that the fringes get fainter further away from the centre. Consequently, not all 15 fringes may be observable.

Section Summary

  • Young’s double slit experiment gave definitive proof of the wave character of light.
  • An interference pattern is obtained by the superposition of light from two slits.
  • There is constructive interference when  d sinθ = mλ  for  m = 0, 1, -1, 2, -2, 3, -3, 4, -4, ….(constructive) , where  d is the distance between the slits,  θ is the angle relative to the incident direction, and  m is the order of the interference.
  • There is destructive interference when  d sinθ =  ( m + 1 / 2 )λ   for  m = 0, 1, -1, 2, -2, 3, -3, 4, -4, ….(constructive)

Conceptual Questions

1: Young’s double slit experiment breaks a single light beam into two sources. Would the same pattern be obtained for two independent sources of light, such as the headlights of a distant car? Explain.

2: Suppose you use the same double slit to perform Young’s double slit experiment in air and then repeat the experiment in water. Do the angles to the same parts of the interference pattern get larger or smaller? Does the colour of the light change? Explain.

3: Is it possible to create a situation in which there is only destructive interference? Explain.

4:   Figure 7 shows the central part of the interference pattern for a pure wavelength of red light projected onto a double slit. The pattern is actually a combination of single slit and double slit interference. Note that the bright spots are evenly spaced. Is this a double slit or single slit characteristic? Note that some of the bright spots are dim on either side of the centre. Is this a single slit or double slit characteristic? Which is smaller, the slit width or the separation between slits? Explain your responses.

The figure shows a photo of a horizontal line of equally spaced red dots of light on a black background. The central dot is the brightest and the dots on either side of center are dimmer. The dot intensity decreases to almost zero after moving six dots to the left or right of center. If you continue to move away from the center, the dot brightness increases slightly, although it does not reach the brightness of the central dot. After moving another six dots, or twelve dots in all, to the left or right of center, there is another nearly invisible dot. If you move even farther from the center, the dot intensity again increases, but it does not reach the level of the previous local maximum. At eighteen dots from the center, there is another nearly invisible dot.

Problems & Exercises

  • At what angle is the first-order maximum for 450-nm wavelength blue light falling on double slits separated by 0.0500 mm ?
  • Calculate the angle for the third-order maximum of 580-nm wavelength yellow light falling on double slits separated by 0.100 mm.
  • What is the separation between two slits for which 610-nm orange light has its first maximum at an angle of 30.0  degrees?
  • Find the distance between two slits that produces the first minimum for 410-nm violet light at an angle of 45.0  degrees.
  •  Calculate the wavelength of light that has its third minimum at an angle of 30.0 o when falling on double slits separated by 3.00 μm. Explicitly, show how you follow the steps in  Problem-Solving Strategies for Wave Optics .
  • What is the wavelength of light falling on double slits separated by 2.00 μm if the third-order maximum is at an angle of 60.0 o ?
  • At what angle is the fourth-order maximum for the situation in Problems & Exercises 1 ?
  • What is the highest-order maximum for 400-nm light falling on double slits separated by 25.0 μm ?
  • Find the largest wavelength of light falling on double slits separated by 1.20 μm for which there is a first-order maximum. Is this in the visible part of the spectrum?
  • What is the smallest separation between two slits that will produce a second-order maximum for 720-nm red light?
  • (a) What is the smallest separation between two slits that will produce a second-order maximum for any visible light? (b) For all visible light.
  • (a) If the first-order maximum for pure-wavelength light falling on a double slit is at an angle of 10.0o, at what angle is the second-order maximum? (b) What is the angle of the first minimum? (c) What is the highest-order maximum possible here?
  • Figure 8 shows a double slit located a distance x from a screen, with the distance from the centre of the screen given by y.  When the distance d between the slits is relatively large, there will be numerous bright spots, called fringes. Show that, for small angles (where sinθ≈tanθ, with θ in radians), the distance between fringes is given by Δy = x λ /d.

The figure shows a schematic of a double slit experiment. A double slit is at the left and a screen is at the right. The slits are separated by a distance d. From the midpoint between the slits, a horizontal line labeled x extends to the screen. From the same point, a line angled upward at an angle theta above the horizontal also extends to the screen. The distance between where the horizontal line hits the screen and where the angled line hits the screen is marked y, and the distance between adjacent fringes is given by delta y, which equals x times lambda over d.

14: Using the result of the problem above, calculate the distance between fringes for 633-nm light falling on double slits separated by 0.0800 mm, located 3.00 m from a screen as in Figure 8 .

15: Using the result of the problem two problems prior, find the wavelength of light that produces fringes 7.50 mm apart on a screen 2.00 m from double slits separated by 0.120 mm (see Figure 8 ).

1:  0.516 o

3:  1.22 x10 -6 m

4: 0.290 μm

9: 1200 nm (not visible)

10 :  1.44 μm

11: (a) 760 nm (b) 1520 nm

13: For small angles sinθ ≈ tanθ≈θ when the angle is in radians

For two adjacent fringes we have d sinθ m = m λ

and d sinθ m +1 =  (m+1)  λ

Subtracting these equations gives

14: 2.37 cm

Douglas College Physics 1207 Copyright © August 22, 2016 by OpenStax is licensed under a Creative Commons Attribution 4.0 International License , except where otherwise noted.

Share This Book

We are familiar with the double-slit experiment using , and the pattern it produces. We consider the same experiment but with a physical particle (usually an electron) in the low-intensity limit. Experimentally we see that even though only a single electron at a time passes through the system, an interference pattern still appears.

This means that the underlying de Broglie wave of the particle interferes with itself, even though we consider only a single particle to be in the system at a time.

Consider a setup like shown, where a laser is shined at the particle as it passes through one of the slits. A microscope records the scattered photons and can determine which slit the particle passed through depending on the scattering angle.

The red waves show photons, while the black waves show the incident particle. The dashed lines show the path to a far-off screen where one would expect interference.

What we see experimentally is that if we attempt to measure the slit passed through in this way, the interference pattern disappears. Shining a laser on the particle's path at all, even if the scattering is not recorded, destroys the interference pattern.

Consider lowering the intensity of the laser such that on the order of a single photon is emitted at a time. What we see is that we recover a partial interference pattern albeit with incomplete contrast.

The black line shows an interference pattern with complete contrast, scaled from 0 to 1. The blue line has incomplete contrast, the lows don't reach 0 and the highs don't reach 1.

Instead, we might lower the frequency of the photons so they carry less energy and interact weakly with the particle. If the frequency is low enough we again recover the interference pattern, but when λd we lose the optical resolution to determine which slit a photon was scattered from.

Any way the information is available, we cannot know both the position of the particle and maintain an interference pattern.

Consider the wave functions at the two slits ψ1=Aeiϕ1 and ψ2=Aeiϕ2, with ϕ1=ϕ2 initially. At the wall (call it point D) where we expect interference, the wave function ψD=AeikL1+AeikL2 where k is the wavenumber and L is the length traveled by each wave. This gives us probability

ψD2=2∣A2[1+cos(k(L1L2))].

Now consider the behavior when a photon interacts with the particle.

When the photon and electron collide the photon imparts some momentum upon the electron. This corresponds to a change in phase of wave vector such that ψiψiei(kinksc)xi. Here x1,x2 are the positions of the slits; we take the dot product to adjust for the phase difference of the incident laser. This gives us the probability

ψD2=2∣A2 1+cos(k(L1L2)+phase shift (kinksc)(x1x2)) .

We see that the only non-constant term is ksc. Since the scattering angle is random, the phase shift will be random too. These random phases cause the interference patterns to disappear. However, if we know the scattering angle of the photon that collided with a given electron, we can reconstruct the interference pattern by adjusting for the phase difference.

This also explains the behavior the incident wavelenght is high. k=λ2π, so when λ is large, kinksc is small, as is the phase shift.

Plus.Maths.org

icon

Physics in a minute: The double slit experiment

double slit experiment quora

One of the most famous experiments in physics is the double slit experiment. It demonstrates, with unparalleled strangeness, that little particles of matter have something of a wave about them, and suggests that the very act of observing a particle has a dramatic effect on its behaviour.

To start off, imagine a wall with two slits in it. Imagine throwing tennis balls at the wall. Some will bounce off the wall, but some will travel through the slits. If there's another wall behind the first, the tennis balls that have travelled through the slits will hit it. If you mark all the spots where a ball has hit the second wall, what do you expect to see? That's right. Two strips of marks roughly the same shape as the slits.

In the image below, the first wall is shown from the top, and the second wall is shown from the front.

Double slit

The pattern you get from particles.

Now imagine shining a light (of a single colour, that is, of a single wavelength) at a wall with two slits (where the distance between the slits is roughly the same as the light's wavelength). In the image below, we show the light wave and the wall from the top. The blue lines represent the peaks of the wave. As the wave passes though both slits, it essentially splits into two new waves, each spreading out from one of the slits. These two waves then interfere with each other. At some points, where a peak meets a trough, they will cancel each other out. And at others, where peak meets peak (that's where the blue curves cross in the diagram), they will reinforce each other. Places where the waves reinforce each other give the brightest light. When the light meets a second wall placed behind the first, you will see a stripy pattern, called an interference pattern . The bright stripes come from the waves reinforcing each other.

Double slit

An interference pattern.

Here is a picture of a real interference pattern. There are more stripes because the picture captures more detail than our diagram. (For the sake of correctness, we should say that the image also shows a diffraction pattern , which you would get from a single slit, but we won't go into this here, and you don't need to think about it.)

Double slit

Image: Jordgette , CC BY-SA 3.0 .

Now let's go into the quantum realm. Imagine firing electrons at our wall with the two slits, but block one of those slits off for the moment. You'll find that some of the electrons will pass through the open slit and strike the second wall just as tennis balls would: the spots they arrive at form a strip roughly the same shape as the slit.

Now open the second slit. You'd expect two rectangular strips on the second wall, as with the tennis balls, but what you actually see is very different: the spots where electrons hit build up to replicate the interference pattern from a wave.

Double slit

Here is an image of a real double slit experiment with electrons. The individual pictures show the pattern you get on the second wall as more and more electrons are fired. The result is a stripy interference pattern.

Double slit

Image: Dr. Tonomura and Belsazar , CC BY-SA 3.0

How can this be?

One possibility might be that the electrons somehow interfere with each other, so they don't arrive in the same places they would if they were alone. However, the interference pattern remains even when you fire the electrons one by one, so that they have no chance of interfering. Strangely, each individual electron contributes one dot to an overall pattern that looks like the interference pattern of a wave.

Could it be that each electrons somehow splits, passes through both slits at once, interferes with itself, and then recombines to meet the second screen as a single, localised particle?

To find out, you might place a detector by the slits, to see which slit an electron passes through. And that's the really weird bit. If you do that, then the pattern on the detector screen turns into the particle pattern of two strips, as seen in the first picture above! The interference pattern disappears. Somehow, the very act of looking makes sure that the electrons travel like well-behaved little tennis balls. It's as if they knew they were being spied on and decided not to be caught in the act of performing weird quantum shenanigans.

What does the experiment tell us? It suggests that what we call "particles", such as electrons, somehow combine characteristics of particles and characteristics of waves. That's the famous wave particle duality of quantum mechanics. It also suggests that the act of observing, of measuring, a quantum system has a profound effect on the system. The question of exactly how that happens constitutes the measurement problem of quantum mechanics.

Further reading

  • For an extremely gentle introduction to some of the strange aspects of quantum mechanics, read Watch and learn .
  • For a gentle introduction to quantum mechanics, read A ridiculously short introduction to some very basic quantum mechanics .
  • For a more detailed, but still reasonably gentle, introduction to quantum mechanics, read Schrödinger's equation — what is it?

Originally published on 05/02/2017.

  • Add new comment

Double slit experiment

The scientist at Washington University found that quasimeasurements cause the zeno effect possibly explaining why the particles do not form a interference pattern if one detects which slit they pass through.

double slit experiment

Seem to be leaving out the fact that the difference occurs when being actively observed.

Heisenberg uncertainty principle

Everything we see is our brain "interpreting" the photons of light reflected off a object. Just like our brains turns 30 FPS and up into a smooth video image. Any experiment that has the word "Observation" in it is flawed. A human used as test equipment for the observation part of a experiment can never be accurate.

It has nothing to do with a human observing anything. It has to do with how one observes things at the atomic and quantum scale. We make these observations by bouncing other particles off of the particles we're interested in examining. At the macro-scale this is not a problem as the particles were bounce off of things are much smaller and have little no affects at the macro.

But at the atomic and smaller scales, the particles we bounce off of things to observe them are similar in "size" (this is a stand in for mass, charge, etc.) to the particles we are trying to observe.

You can think of it like trying to figure out where a billiard ball is by bouncing a golf ball it. That will change the position, spin, etc. of the billiard ball.

I agree with you. Yet, seemingly, the rest of the world either believes - when they do not believe what you've just said - humans have a psychic grip on subatomic particles, or this proves a God exists.

Why not both?

I believe both, and agree with both of you. The two dont have to be mutually exclusive... It actually makes for a more narrow view that way.

Leave room for more

I agree. What we don't know is much greater than what we do and much of what we think we know will change. I think it's best to leave room for many possibilities. Magic is just science undiscovered. If we keep placing boundaries on what's possible and teach others to ignore something for lack of explanations, scientific discovery suffers.

The two slit experiment

The duality of the particle has nothing to do with proving a god exists, just that science is indeterminate and is a duality of possible existences dependent on the observation of a consciousness. Seems like its human consciousness that determines the outcome not any god.

This isn't actually true either. Experiments have shown that even if the photon used to make an observation is of low enough energy that it doesn't alter a large molecules trajectory much, the interference pattern still disappears.

Also, null interaction experiments have been done in which there is no contact between particles at all. But if info about which slit can be found, the interference pattern disappears.

Source for that?

Hey, do you have a source on finding information about which slit with out active observation causing the interference pattern to not appear?

Information on the Double Split experiment

Try QED: The Strange Theory of Light and Matter by Richard P. Feynman. It's been a while since I've enjoyed this book, but my recollection is that it covered the topic well.

Thank you! I didn’t think that explanation made sense, since any effect upon the particle being observed would surely be taken into account in these experiments…and because the detection unit (which “catches” the electrons passing through either slit) doesn’t work by shooting particles at them, as far as I know…even if it did, that wouldn’t be ignored as a variable or whatever…right? I assume such interaction isn’t the method of detection anyway; how the materials used in the experiment could potentially influence the subject being measured is exactly what they control for, among other things- the environment itself, the actions taken as it is conducted and how conditions change…etc. I just don’t think what the commenter described is true since, well, I’d assume the researchers would know that sort of thing could skew the results & therefore lead to an incorrect conclusion. Scientists aren’t just straight up missing the impact of what would be such an obvious flaw in these experiments. I mean, in general they either eliminate the possibility of their tools affecting what they’re measuring OR they take that into account as a variable. Usually the second one is only possible if it’s something that like…as long as it’s known, it won’t render their data useless…if that makes sense (so being aware that it is a factor is key) Anyway…

yet this works at the molecular level too

Molecules are much larger than photons, yet you get the same result.

Quantum Games

If a player has two attached low emission lasers either side of head, beamed through a double slit screen at, say, a home movie or scenario created by the player, bounces back as photons via player's retina to the player's neurons, will player perceive or believe he/she is part of the home movie?

Are you Sure?

Placing only 1 detector in front of one of the double slits ALSO collapses the wave function of both slits. This unequivocally proves that it isn’t the measurement method, but the ACT of measurement itself.

For example, we get the wave pattern. We place a detector in front of both holes we then get 2 bands. ..now, if it were the detector interfering as you mentioned, we will take 1 detector away and leave the other. This way only 1 slit has a detector that interacts with the electron as it passes through.

This would mean the slit with the detector produces a band while the slit without the detector produces a partial striped pattern of the wave.

This is not what we observe however. Measuring just 1 slit still causes 2 bands.

This is because even measuring just 1 slit gives us information on the other. It’s the information that is seemingly the cause of the collapse of the wave function.

Process of Elimination

Hello, I like your idea of removing one detector to see if there is a tangible difference. Given how complex the science behind this is, it doesn't seem ethical to have a biased conclusion with such conviction after just 1 adjustment.

Why has no one tried 3,4,5 slits? With those slits, why haven't we tried removing one detector at a time, and swapping out different ones for each slit? Speeding up or slowing down the particle beam? There are so many variables that could lead to comprehensible evidence if the results are as consistent to previous attempts.

However, it just seems this experiment was done 60 years ago and then we just left it as is with no additional input or further experimentation; just copy paste imitations doing the same thing for educational purposes of the original experiment. That just becomes a history lesson, not a science one.

I come here after watching a conspiracy theory and they referenced this experiment as the leading evidence that we live in a simulation, that the universe is just a projection controlled by a program that detects when its being observed like when a video game detects the players position and renders in whats necessary around them. Suggesting that the light particles can bend the rules of physics and time as soon as someone attempts to measure them.

My initial reaction is not to believe this, however I fully appreciate a rebuttal to come armed with objective, comprehensible evidence like any good debate where possible.

I think this experiment needs a make over and we need to breathe new life into it with far more variables to play with, leaving no stone unturned to draw a general trend and any potential outliers to help solve this once and for all. Or at least, as close as our minds will allow us.

We are our own limitations.

Re: Process of Elimination

I agree with you, the whole furore around this experiment has made it more of a history lesson. It's become almost like the Galileo stone/feather thing; i've had people tell me that in normal every-day circumstances, if i dropped a pen and a pebble at exactly the same moment, they would hit the floor at the same time. Even, when i demonstrate this live, people are still adamant in their stance; after-all, i'm no better than Galileo.

This represents a great discrepancy to the original format of experiment by Galileo. I think some of these experiments have been so popularised in pop-culture and pop-science that people have ruled out the possibility of questioning them. Just like the Galileo stone and feather thing, Everyone is still quoting the same concept from decades ago without any iota of desire to question it. This means that some of these science-cum-history concepts are left to grow into "unquestionables" filled with errors.

I think its about time we re-visit the whole premise of this experiment. Let's introduce 3,4, or even 10 slits! Let's do it today with more control over the variables. We can't let this become another one of the "unquestionables". We can't adopt beliefs and never question them. That would be disastrously dogmatic. The opposite of what it means to do real science!

Also, what was the youtube video you watched? thanks

Uh...no. That is definitely not what happens. If so, there would be no conceptual problem. But don't take my word for it. Here's what Richard Feynman said: “I think I can safely say that nobody really understands quantum mechanics." That's because QM seems to suggest that there's a real connection between the mind of the observer and the results obtained. Also, you're not taking into account the results of the quantum eraser phenomenon, which is another aspect of this experiment that suggests the trajectory of an electron in the past can be altered by an experimenter's actions in the present. You'll have to look it up as it's rather lengthy.

Impact of measuring

Thank you so much for this explanation! We have been fretting about this for quite a while. I wish this physical interaction were more clearly included in other explanations of the slit experiment or just quantum mechanics and general

Question for you

You’re saying to observe a particle we’re bouncing particles off of that observed particle? I don’t think that’s the case. What exactly is bouncing off the particle being observed? How is it being directed toward the target particle? With the way they design these experiments, as far as I know, there should be no overt effect like that- certainly not the actual impact of matter as you’re describing. I don’t think anything is being expelled from the detection materials. Or if it were, that would be taken into account- so precise calculations would be made about how it should impact the results. Basically, if something physical was being intentionally shot at the particles and that somehow was the way we detected them, then the scientists performing the research would do that math using specific measurements (including the mass of that projectile matter). I mean if there was anything being directed toward the electrons or whatever, they’d surely have an idea of what forces it would exert and the interaction it should have, etc… Now you could say the electron being observed has some effect upon the detection unit itself (the “quantum observer”) because logically, in order to even register the electron’s presence/position, it must. But I believe that by all known science, there shouldn’t be any effect upon the particle being observed- other than the fact that it’s being observed. That’s kind of the whole point and is exactly what makes this discovery so mind-blowing…right? So I would assume that in these experiments, they’re controlling for those conditions (the observation device having any physical effect or exerting force upon the observed electron, and all possible variables). Do you disagree? I am genuinely curious about what you’re asserting!

What if there are smaller units than photons, which are presently invisible to our instruments? We'll call them units of consciousness (or thought) that we as conscious beings emanate without knowing it, and it is these very tiny units of consciousness (relative to the photon) that influence the photons to behave as they do. In other words the invisible is influencing the visible like the soul influences the body. These same units could explain the placebo effect.

curtesy call

I agree to your theory. Just want to point it out there as a suggestion to proof read your work before submitting it. I have I found a couple of mistakes where a word had been left out.

Curtesy call

I thought it was well thought out and written. Has it been edited since your comment? If we're being semantic you may wish to check your spelling of courtesy.

Objective observation, unlike subjective observation, is very much allowed in conducting data in experiments.

Plus, unless you have robots conducting the experiment ans/or collecting the data, humans are going to be involved.

observer does not need to be human!

A houseplant would work too. Humans cannot see at this level in any case. Machinery is used, and humans don't have to be in the room for the effects to continue. Observation just means measurement, and Wheeler's Delayed Choice and the Quantum Eraser experiments showed the measurement can occur after the photon, electron, or molecule has hit the wall...and it will still change.

This word is used for convenience, but no conscious observer is required. You can also say “detected”. And by detected what is meant is that information exists that is, in principal, detectable even if not yet technically feasible. Look up “The World’s Smallest Double-Slit Experiment” (2007) and you will find that a single low-energy electron can be an “observer” and collapse the quantum interference pattern of a high-energy electron exiting a single hydrogen molecule.

This is BS. When being observed by a sensor, electrons behave as particles. You lack sufficient understanding of the double slit experiment.

A summarily dismissal of a complex phenomenon that has world class physicists, accompanied by a " You don't understand" line illustrates a simple mind

Deeper understanding is to look at our process of observation.

Everything we recieve through our sences gets interpreted in our brains as being solid, founded by rules/laws/logic & most importantly being OUTSIDE our bodies, ie, being real. Reality is though, that these words you now read, are IN your head as is ALL experiences. Point being, WHY are we being TRICKED to think we are experiencing life OUTSIDE our heads when in fact, we are experiencing life IN our heads, just a observation.

Slit Experiment

The only reason a human may not be good test equipment for observation would be because the person lacks awareness. Human observation can be as accurate as any mechanical scientific devise. It just depends on the awareness of the individual.

by detector , they mean, not human but mechanical

Double Split Experiment

"Seem to be leaving out the fact that the difference occurs when being actively observed" EXACTLY!! This experiment shows that matter is not what we think it is. Scientists have known this for a century yet scientific materialism for some reason still prevails. Matter is a product of Mind. NOT the other way around. For more information read "Ontological Mathematics"

That is not what comes out

That is not what comes out this. "Observer" is a misleading term. It does not specifically refer to humans, nor even conscious creatures, although they can be.

About what you wrote

I do agree with you

double slit Hijinx

Ive noticed this trend everywhere. I chock it up to the materialists clinging to their dead ideology

What happens as the distance between the slits becomes greater ? Is there a relationship between the distance between the slits, and the distance of the source from the slits??

Double slit

What if the light is reacting to the material the slits were cut out from. Maybe electromagnetism causing the light particles to bend and change their direction just like how planets and comets change their orbits when passing near something with mass.

The double slits, are on the

The double slits, are on the both sides of the direction of the flow of light. Also, the slit will be more massive on the two outer edges of the double slits. If it were to be hindered due to presence of slits, wouldn't the effect be more on the outer edges and not on the inner edges, i.e. in the middle.

Double Slit

Such is the problem. Particles don't bend when acted upon by electromagnetic fields. They simply form a trajectory. Bending or warping is the property of a wave. Their trajectory could be altered but I'm sure the material is neutral in all aspects to avoid interference.

Electrons have almost no mass and therefore almost no gravity. Atoms of the slit have a huge mass compared to the electrons. As the electrons passes the slit the gravity of the atoms cause some of the closest electrons to start to spin, the same way water spins when shot through a slit. This spin then sends some of the electrons out of their normal straight line trajectory which causes the apparent wave effect.

If that were the case it would happen if there was only one slit. But it doesn't.

Doesn't bend when "observed".

Electrons spin.

Electrons always have a spin of 1/2. This is a fundemental property of electrons and all fermions. Even if electrons were pushed off their trajectory, how do electrons shot one at a time form an interference pattern?

Because it behaves like wave regardless by itself or by groups

When you say wave, do you mean as in bye bye?

Running paint

If light passes through two slits and it reflects off a object that photon leaves some of itself behind and continues on as if you put paint on your hand and slap a wall then run, how many walls you slap depends on how much paint you got, when light reflects off the object or the slit where does it go is it passing through itself or is it colliding with itself in that direction and handing itself some extra light to continue like say you been running with paint on your hand and you have a train of people that follow you and you are the leader. What if you grab just a finger swap of paint for that extra inch foot mile etc. so when the rays reflects off the rectangle will that rectangle of photons continue colliding into each other and handing itself more photons or snatching some to create a another rectangle and so on till it fades away.

agree with Chris Isaacson

I also thought the materials used may have properties that interfere, and the detector might too.

How does the detector itself work? Does it not rely on an intrinsic property of electrons to function? Connecting the detector to that property then necessarily interferes with the experiment. That interference is then to be expected and can be explained rationally rather than through a spooky effect, quantum effect.

Light/Photon/Electron/Particle interacting with slit material

Agree with you. Details of double/single slit experiments should give more information on the slits material, their dimensions (gap width and distance between slits) and the probability of these materials influencing the path/direction of WAVICLES (waves-cum-particles!). This topic needs to discussed and debated.

Getty / Futurism

Former NASA Scientist Doing Experiment to Prove We Live in a Simulation

Did we really take the red pill, the blue pill.

Could we be trapped inside a simulated reality, rather than the physical universe we usually assume?

It's a tantalizing theory, long theorized by philosophers and popularized by the 1999 blockbuster "The Matrix." What if there was a way to find out once and for all if we're living inside a computer?

A former NASA physicist named Thomas Campbell has taken it upon himself to do just that. He devised several experiments, as detailed in a 2017 paper published in the journal The International Journal of Quantum Foundations , designed to detect if something is rendering the world around us like a video game.

Now, scientists at the California State Polytechnic University (CalPoly) have gotten started on the first experiment, putting Campbell's far-fetched hypothesis to the test.

And Campbell has set up an entire non-profit called Center for the Unification of Science and Consciousness (CUSAC) to fund these endeavors. The experiments are "expected to provide strong scientific evidence that we live in a computer-simulated virtual reality," according to a press release by the group.

Needless to say, it's an eyebrow-raising project. As always, extraordinary claims will require extraordinary evidence — but regardless, it's a fun idea.

Simulation Hypothesis

Campbell's experiments include a new spin on the double-slit experiment, a physics demonstration designed to show how light and matter can act like both waves and particles.

Campbell believes that by removing the observer from these experiments, the actual recorded information never existed in the first place. That's instead of current quantum physics suggesting the existence of entanglement that links particles across a distance.

In simple terms, without a player, the universe around them doesn't exist, much like a video game — proof, in Campbell's thinking , that the universe is exclusively "participatory."

Campbell isn't the first to explore a simulation hypothesis. Back in 2003, Swedish philosopher Nick Bostrom published a paper titled " Are You Living in a Computer Simulation? "

Basically, his idea was that if we progress far enough technologically, we'll probably end up running a simulation of our ancestors. Give those simulated ancestors enough time, and they'll end up simulating their own ancestors. Eventually, most minds in existence will be inside layers of simulations — meaning that we probably are too.

Campbell's hypothesis takes a different tack than Bostrom's "ancestor simulation," arguing that our "consciousness is not a product of the simulation — it is fundamental to reality," in CUSAC's press release.

If he were to be successful in his bid to prove that humanity is trapped in a virtual reality — an endeavor that would subvert our basic understanding of the world around us — it could have major implications.

Campbell argued that the five experiments could "challenge the conventional understanding of reality and uncover profound connections between consciousness and the cosmos."

More on the simulation hypothesis: Famous Hacker Thinks We're Living in Simulation, Wants to Escape

Share This Article

COMMENTS

  1. Is there a consensus on the double-slit experiment? : r/AskPhysics

    The double-slit experiment is a nice illustration of wave interference. The only thing that is debated is the interpretation of wave function collapse (in the case those waves are quantized), but that is not specific to the double-slit experiment. ... but looking at Quora as a layperson (as the person who asked me to explain it did) it would ...

  2. The double-slit experiment: Is light a wave or a particle?

    When Young first carried out the double-split experiment in 1801 he found that light behaved like a wave. Firstly, if we were to shine a light on a wall with two parallel slits — and for the ...

  3. Double slit experiment outcome changing by observing the slits

    The general rule for double slit interference is: There will be interference UNLESS there is the possibility, in principle, for determining which-slit information. It does not matter whether you know which slit the particle goes through, it is enough to eliminate the interference that you could have obtained this information.

  4. Double-slit Experiment

    The double-slit experiment is an experiment in quantum mechanics and optics demonstrating the wave-particle duality of electrons, photons, and other fundamental objects in physics. When streams of particles such as electrons or photons pass through two narrow adjacent slits to hit a detector screen on the other side, they don't form clusters based on whether they passed through one slit ...

  5. Double-slit experiment

    In modern physics, the double-slit experiment demonstrates that light and matter can satisfy the seemingly incongruous classical definitions for both waves and particles. This ambiguity is considered evidence for the fundamentally probabilistic nature of quantum mechanics.This type of experiment was first performed by Thomas Young in 1801, as a demonstration of the wave behavior of visible ...

  6. Finally making sense of the double-slit experiment

    Feynman stated that the double-slit experiment "…has in it the heart of quantum mechanics. In reality, it contains the only mystery" and that "nobody can give you a deeper explanation of this phenomenon than I have given; that is, a description of it" [Feynman R, Leighton R, Sands M (1965) The Feynman Lectures on Physics].We rise to the challenge with an alternative to the wave ...

  7. The double-slit experiment with a pit in the screen

    The main question is contained below in the experiment #6. But first, 5 simple experiments are described to describe the designations that are used in the question of the experiment #6. In the double-slit experiment, the slits are labeled α and β. A region C is designated on the screen.

  8. The double-slit experiment

    Young carried out his original double-slit experiment with light some time in the first decade of the 1800s, showing that the waves of light from the two slits interfered to produce a characteristic fringe pattern on a screen. In 1909 Geoffrey Ingram (G I) Taylor conducted an experiment in which he showed that even the feeblest light source ...

  9. What is the double-slit experiment, and why is it so important?

    The more individual photons you shoot through the double slit, the closer that photon detector comes to detecting photons 50% of the time, just as flipping a coin 10 times might give you heads 70% ...

  10. PDF The Double-Slit Experiment: An Adventure in Three Acts

    The double-slit experiment throws into stark relief two of the most enduring enigmas about quantum mechanics: the role of probabilities, and the strange intermixing of particle and wave concepts ("wave-particle duality"). We will begin by considering two separate classical scenarios: firing macroscopic bullets at a wall, and watching an ...

  11. Two slits and one hell of a quantum conundrum

    All in all, the double-slit experiment seems, to borrow from Feynman again, "screwy". The original experiment, as Ananthaswamy notes, was classical, conducted by British polymath Thomas Young ...

  12. Double Slit Experiment, how does manipulating the type of ...

    From what I understand, they always act as a wave. See Sabine's video on the quantum eraser , in the first few minutes of that video she discusses the double slit and debunks a common misconception. It's still a wave / still produces a diffraction pattern, even when it goes through one slit. It's just a one slit diffraction pattern.

  13. Young's Double Slit Experiment: A Simple Explanation

    The double-slit experiment shows the duality of the quantum world. A photon's wave/particle duality is affected when it is observed. Light has been one of the major areas of inquiry for physicists since we first began questioning the world around us. Understandably so, as it is the medium by which we see, measure and understand the world.

  14. 3.4 Young's Double Slit Experiment

    Douglas College Physics 1207. 3.4 Young's Double Slit Experiment. λSummary. Explain the phenomena of interference. Define constructive interference for a double slit and destructive interference for a double slit. Although Christiaan Huygens thought that light was a wave, Isaac Newton did not. Newton felt that there were other explanations ...

  15. The Double-Slit Experiment: Quantum Mechanics Explained

    Explore the groundbreaking Double-Slit Experiment and its profound implications for quantum mechanics. Learn how this experiment challenged classical physics...

  16. How do we know the double slit experiment wasn't just ...

    First, the experiment has been replicated many times in many different ways: lasers, electron beams, neutrons, helium atoms scattering off of deuterium atoms, and more. Second, all of chemistry is a form of the double-slit experiment. Just as the wave function has destructive interference in the dark spots on the screen, all fermions exhibit ...

  17. Double slit experiment

    Double slit experiment. We are familiar with the double-slit experiment using light, and the interference pattern it produces. We consider the same experiment but with a physical particle (usually an electron) in the low-intensity limit. Experimentally we see that even though only a single electron at a time passes through the system, an ...

  18. Physics in a minute: The double slit experiment

    Submitted by Marianne on 19 November, 2020. One of the most famous experiments in physics is the double slit experiment. It demonstrates, with unparalleled strangeness, that little particles of matter have something of a wave about them, and suggests that the very act of observing a particle has a dramatic effect on its behaviour.

  19. Former NASA Scientist Doing Experiment to Prove We Live in a ...

    Campbell's experiments include a new spin on the double-slit experiment, a physics demonstration designed to show how light and matter can act like both waves and particles.

  20. Young double slit experiment Hindi || Quantum Physics

    Hello friends in this video we will talk about the famous Thomas young double slit experiment we is regarded as a milestone in physics and is among the 10 ...

  21. In Young's double slit experiment, the separation of

    Question: In Young's double slit experiment, the separation of four bright fringes is 2.5 mm.The wavelength of light used is 6.2 times 10 ^-5 cm. Calculate the separation of slits.[Ans, d = 7.94 \times 10 -4 m]